Sunteți pe pagina 1din 287

Dpto.

de Econom a Cuantitativa
Universidad Complutense de Madrid

Introducci on a la Econometr a
Tema 2 Probabilidad multivariante Marcos Bujosa y Gustavo A. Marrero

Material de apoyo para el curso Introducci on a la Econometr a de la licenciatura en Econom a de la Universidad Complutense de Madrid.

c 20032007 Marcos Bujosa y Gustavo A. Marrero Actualizado el: 1 de junio de 2007

Versi on 4.1

Copyright c 20032007 Marcos Bujosa y Gustavo A. Marrero

Algunos derechos reservados. Esta obra est a bajo una licencia Reconocimiento-CompartirIgua de Creative Commons. Para ver una copia de esta licencia, visite http://creativecommons. org/licenses/by-sa/2.5/es/deed.es o env e una carta a Creative Commons, 559 Nathan Abbott Way, Stanford, California 94305, USA.

Puede encontrar la u ltima versi on de este material en: http://www.ucm.es/info/ecocuan/mbb/index.html#ietria

Indice Indice 3

Probabilidad multivariante
1. Introducci on 2. Espacio de probabilidad 2.1. Probabilidad condicionada . . . . . . . . . . . . . . . . . . 3. Variables aleatorias 3.1. Variables aleatorias discretas . . . . . . . . . . . . . . . . 3.2. Variables aleatorias continuas . . . . . . . . . . . . . . . . 4. Variables aleatorias bivariantes

8
9 21 26 34 36 40 51

5. Distribuciones marginales 6. Distribuciones condicionadas 6.1. Caso discreto . . . . . . . . . . . . . . . . . . . . . . . . 6.2. Caso continuo . . . . . . . . . . . . . . . . . . . . . . . . 7. Independencia

76 84 85 98 113

8. Momentos conjuntos 118 8.1. Propiedades de los momentos bivariantes . . . . . . . . . . 122 9. Momentos condicionados 139 9.1. Propiedades de la esperanza condicional . . . . . . . . . . 144 10.Funci on de regresi on y funci on ced astica. Esperanzas y varianzas condicionales estoc asticas 160 10.1. Esperanzas iteradas . . . . . . . . . . . . . . . . . . . . . 167 10.2. Identidad de la varianza condicional . . . . . . . . . . . . 169

10.3. Propiedades de la esperanza condicional estoc astica . . 10.4. Esperanza condicional cuando es una funci on lineal . . 10.5. Aproximaci on lineal de la esperanza condicional. Recta regresi on . . . . . . . . . . . . . . . . . . . . . . . . . 11.Transformaci on de variables 12.Distribuciones multivariantes 13.Funci on generatriz de momentos 14.Preguntas y problemas 15.Trasparencias 16.Bibliograf a A. Momentos univariantes

. . . . de . .

180 184 187 201 222 227 243 269 273 275

B. Demostraciones . Soluciones a los Ejercicios

278 281

Este es un material de apoyo a las clases. En ning un caso sustituye a los libros de texto que guran en el programa de la asignatura; textos que el alumno debe estudiar para afrontar el examen nal con ciertas garant as de exito. El programa se cubre con los siguientes cap tulos de libro de texto Novales (1997)1 : Cap tulos 1 a 3: Estos temas han sido cubiertos en asignaturas anteriores, y debido a su bajo nivel de complejidad no se ver an en clase (aunque forman parte del programa). Cap tulos 4 a 6: Estos temas han sido cubiertos en las asignaturas Estad stica I y II. Se realizar a un breve repaso en clase (una semana o semana y media como m aximo), asumiendo que el alumno es capaz de preparar por su cuenta esta parte. Cap tulos 7 y 8: completos Cap tulo 9: secciones 9.4 a 9.6 Cap tulos 10 y 12: completos

1 Otros excelentes manuales en castellano son Pe na (2001), Pe na (2002) y Pe na y Romo (1997).

Probabilidad multivariante

1. Introducci on Una buena introducci on aparece en (Spanos, 1999, Cap. 1).

Modelo Te orico vs Mundo Real

Cuanto mide una linea pintada sobre la diagonal de una pared? (mundo real)

A.- Teor a matem atica (Geometr a Eucl deaTeorema de Pit agoras) B.- Modelo te orico (representaci on abstracta de la pared)

Del modelo podemos inferir una respuesta tentativa midiendo los lados

A2 = B 2 + C 2
A

A=
B

B2 + C 2

Regularidades en el azar

Azar: incertidumbre sobre cada resultado particular de un fen omeno Regularidad: persistente regularidad en comportamiento de resultados Regularidad en el azara 1. Distribuci on 2. Dependencia (o Independencia) 3. Heterogeneidad (u Homogeneidadb )

Lanzamiento de un dado pl astico o plastilina?


a (v ease

octave

b Id entica

Spanos, 1999, Capitulo 1) distribuci on (ID)

1 2 3 4 5 6

1 (1,1) (2,1) (3,1) (4,1) (5,1) (6,1)

2 (1,2) (2,2) (3,2) (4,2) (5,2) (6,2)

3 (1,3) (2,3) (3,3) (4,3) (5,3) (6,3)

4 (1,4) (2,4) (3,4) (4,4) (5,4) (6,4)

5 (1,5) (2,5) (3,5) (4,5) (5,5) (6,5)

6 (1,6) (2,6) (3,6) (4,6) (5,6) (6,6)

Cuadro 1: resultados elementales en el lanzamiento de dos dados

Lanzamiento de dos dados

Distribuci on, Independencia, Homogeneidad


12 10 8 6 4 2 20 40 60 80 100 120 140 160 180 200
octave

Suma de dos dados

Lanzamiento de dos dados

Distribuci on triangular
35 30 25 20 15 10 5 Suma de dos dados

10

12
octave

Regularidades en fen omenos econ omicos?

Lanzamiento 2 dados: Distribuci on (triangular) Independencia, Homogeneidad (I.I.D.) Regularidad en fen omenos econ omicos ? No podemos derivar una dist. a priori
(ej.: mediante argumentos de simetr a)

Con frecuencia resultados dependientes; y condiciones cambian con el tiempo (Heteregoneidad). Indice de cotizaciones NASDAQ: Distribuci on (campaniforme para la tasa de variaci on), Dependencia, Heterogeneidad

Indice NASDAQ

Dependencia
5000 4500 Indice NASDAQ (apertura) 4000 3500 3000 2500 2000 1500 1000 500 0 1986 1988 1990 1992 1994 1996 1998 2000 2002 2004
media, banda de variaci on

Tasa de Variaci on del Indice NASDAQ

Heterogeneidad, Dependencia
0.3 Tasa de variacin ndice NASDAQ 0.2 0.1 0 0.1 0.2 0.3 1986 1988 1990 1992 1994 1996 1998 2000 2002 2004
media, banda de variaci on, clusters

Tasa de Variaci on del Indice NASDAQ

Distribuci on (Campaniforme)
8 7 6 5 4 3 2 1 0 -0.2 -0.1 0 0.1 0.2 Estad stico contraste de normalidad: T. V. NASDAQ N(0.01,0.07) 2 2 = 19.2, valor p 0.00007

Estad stica y Econometr a

Objetivo: identicar patrones de regularidad en datos.

0.1

A.- Teor a matem atica: Probabilidad. Formaliza conceptos de distribuci on Temas 1 a 3 (D), independencia (I) y homogeneidad (H). B.- Modelo estad stico
frecuencias relativas distribuci on (N,U,2 ,. . . ) impredicibilidad independencia (I) o dependencia semejanza en pautas homogeneidad (ID) o heterogeneidad (D) (I) (H)

Descripci on de la dependencia y la heterogeneidad (variables econometr a o bien pasado reciente series temporales)

Inferencia estad stica con Modelo estad stico y los datos. Temas 4 a 7

2. Espacio de probabilidad Una buena introducci on aparece en (Spanos, 1999, Cap. 4). en puede consultar Adem as de los cap tulos 4 a 7 de Novales (1997), tambi Pe na (2001, Capitulos 4 y 6) L opez Cachero (1992, Capitulos 17 a 20) Mittelhammer (1996, Capitulos 1 a 3)

Supongamos el siguiente experimento aleatorio (v ease. . . ) Ejemplo 1. E : Lanzamiento de una moneda dos veces. Sucesos de inter es: cualquier resultado: n o n o n o n o ( , ) , ( , ) , ( , ) , ( , ) Espacio de sucesos (y, o, y no): n o n ( , ) , ( , B= n o ( , ), ( , ) , n o ( , ), ( , ) , n ( , ), ( , ), ( n ( , ), ( , ), (

o n ) , ( , n ( , ), ( n ( , ), ( o n , ) , ( o n , ) , (

o n o ) , ( , ) , o n o , ) , ( , ), ( , ) , o n o , ) , ( , ), ( , ) , o , ), ( , ), ( , ) , o , ), ( , ), ( , ) ,

S (que ocurra cualquier cosa suceso seguro ) (que no ocurra nada suceso imposible )

Llamamos ley (o funci on) de probabilidad P () a la funci on: P () : B [0, 1] que verica [Axioma 1] P (S ) = 1 [Axioma 2] P (A) 0, para cualquier suceso A B [Axioma 3] Aditividad contable. Para cualquier sucesi on numerable de sucesos Ai B, i = 1, 2, . . . tales que Ai Aj = , para todo i = j, i, j = 1, 2, . . . ! [ X P Ai = P (Ai ) .
i=1 i=1

Ejemplo 2. [Lanzamiento de dos monedas: (continuaci on del Ejemplo 1 en la p agina22)] Experimento aleatorio, E : Lanzamiento de una moneda dos veces. Sucesos de inter es: cualquier resultado. Ley de probabilidad (si la moneda no est a trucada, ie. P ( ) =1/2): 8 > para 0 > > > > > para S 1 > > n o n o n o n o > > > > para ( , ) , ( , ) , ( , ) , ( , ) , 1 /4 > > n o n o n o < para ( , ), ( , ) , ( , ), ( , ) , ( , ), ( , ) , 1/2 (2.1) P ( ) = n o n o n o > > > > para ( , ), ( , ) , ( , ), ( , ) , ( , ), ( , ) , 1/2 > > n o n o > > > > para ( , ), ( , ), ( , ) , ( , ), ( , ), ( , ) , 3/ 4 > > n o n o > > : para ( , ), ( , ), ( , ) , ( , ), ( , ), ( , ) 3/ 4

Llamamos espacio de probabilidad a la terna (S, B, P ()) es decir espacio muestral: contiene resultados elementales del experimento aleatorio E . espacio de sucesos: contiene sucesos de inter es y sus combinaciones (y, o, y no). ley de probabilidad: indica cuan probable es cada suceso contenido en B.

2.1. Probabilidad condicionada

C omo var a el espacio de probabilidad, (S, B, P ()), cuando disponemos de informaci on adicional? Informaci on adicional: A = el primer lanzamiento ha sido cara n o 1. el nuevo espacio muestral es: SA = ( , ), ( , ) 2. el nuevo espacio de sucesos es: n o BA = SA , , ( , ), ( , ) la nueva ley de probabilidad condicionada 8 > 1 para > > > > <0 para PA () = > 1/2 para > > > > :1/2 para

3.

SA B= o ( , ) n o C= ( , ) n

Probabilidad condicional

10

El nuevo espacio de probabilidad es (SA , BA , PA ()) Podemos derivar la nueva ley de probabilidad condicionada a partir de (S, B, P ())?

P (A B ) P (A) para P (A) > 0; donde P () es la ley de probabilidad original. PA (B ) P ( B | A)

(2.2)

De (2.1) tenemos que n o n o P ( , ) P (A B ) o = PA ( , ) = P ( B | A) = = n P (A) P ( , ), ( , ) tal como aparece m as arriba.


1 4 1 2

1 2

Independencia

11

Cuando el conocimiento del suceso A no afecta a las probabilidades del suceso B decimos que A y B son independientes.

Decimos que los sucesos A y B son independientes si: P ( A | B) = P (A) (2.3)

Usando (2.2) podemos deducir que A y B son independientes si: P (A B ) = P (A) P (B ) . (2.4)

Leyes de probabilidad conjunta y marginales

12

Lanzamiento de dos monedas Espacio Muestral producto S (2) = {( , ), ( , ), ( , ), ( , )}

Leyes de probabilidad marginales 1a moneda P () 1/2 1/2

2a moneda P ()

2/3

1/3

Ley de probabilidad conjunta (1a moneda, 2a moneda) P (2) (, ) ( , ) 2/6 ( , ) 1/6 ( , ) 2/6 ( , ) 1/6

Leyes de probabilidad conjunta, marginales y condicionadas

13

P (2) (, ) 1o moneda P2 ()

2a moneda 2/6 2/6 2/3 1/6 1/6 1/3 P1 () 1/2 1/2 1

Son independientes?

Probabilidad conjunta y marginal

14

renta \ edad pobre mediana rico P2 ()

(18 35) 0.20 0.10 0.01

(36 55) 0.10 0.25 0.06

(56 70) 0.15 0.05 0.08

P1 ()

1. Quien soy? 2. Que edad tengo? 3. Que renta tengo?

3. Variables aleatorias 0.6

Concepto de Vble. aleatoria

15

Variable aleatoria: funci on que asigna n umeros a elementos del espacio muestral S , de manera tal que preserva la estructura de B (y, o, y no). X () : S RX X (s) x; tal que Ax {s : tales que X (s) x} B

RX es el soporte de X .

3.1. Variables aleatorias discretas

Funci on de cuant a y funci on de distribuci on

16

Funci on de cuant a: P X (x) = P (s : X (s) = x)

Funci on de distribuci on: FX (b) P (X b) =


xRX xb

P X (x)

(3.1)

Funci on de cuant a: Distribuci on Bernulli

17

P X (x; ) , 0.8 Probabilidad 0.6 0.4 0.2 0

= 0.6

-1

0 x

0.18

octave

Funci on de distribuci on: Distribuci on Bernulli

18

FX (x; ), 0.8 Probabilidad 0.6 0.4 0.2 0

= 0.6

-1

0 x

0.17

octave

3.2. Variables aleatorias continuas

Funci on de densidad y funci on de distribuci on

19

Funci on de distribuci on:

FX (b) = P (X b)

Funci on de densidad: fX (x), es una funci on continua y positiva que verica P (X b) FX (b) =
xRX xb

fX (x) dx

para todo b RX .

Ya que fX (x) continua, tenemos:

dFX(x) dx

= fX (x)

Funci on de densidad

20

P (a < x b) =

b a fX (x)dx

= FX (b) FX (a)

fX (x)


a b

Funci on de densidad: Distribuci on Normal (Gaussiana)

21

1.2 densidad de probabilidad 1.0 0.8 0.6 0.4 0.2 0.0 -4.0

= 0, 2 = 1.0 = 2, 2 = 0.3 = 1, 2 = 2.0

-3.0

-2.0

-1.0

0.0 x

1.0

2.0

3.0

4.0

Ejercicio 1. [Novales (1997, pp 159)] Sea fX (x) = (a) verique que fX (x) es una funci on de densidad Soluci on : Z
0 6

1 2 x , 72

0<x<6

Z fX (x) dx =
0

6 1 2 1 x3 = 1 216 = 1 x dx = 72 72 3 0 72 3 Ejercicio 1

(b) Calcule P (X > 3) Soluci on : Z P (X > 3) =


3 6 6 3

Z fX (x) dx =

6 1 2 1 x3 = 7 x dx = 72 72 3 3 8 Ejercicio 1

(c) Calcule P (X > 5) Soluci on : Z P (X > 5) =


5 6 6 5

Z fX (x) dx =

6 1 2 1 x3 = 91 = 0.421 x dx = 72 72 3 5 216 Ejercicio 1

(d) Si A es el suceso X > 5 y B es el suceso X > 3 calcule P ( A | B). Compare con la probabilidad anterior. Soluci on : P ( A | B) = P (A B ) P (A) 91/216 13 = = = = 0.481 P (B ) P (B ) 7/8 27

Por tanto, saber que X es mayor que 3 aumenta la probabilidad (condicionada a este hecho) de que X > 5. Ejercicio 1

Ejercicio 2. [Novales (1997, pp 167)] Tomado el ejercicio anterior, calcule la funci on de distrubuci on. Soluci on: La funci on de distribuci on es 8 > 0 para a < 0 > > > > > > <R Ra 2 a 1 FX (a) = 0 fX (x) dx = 72 0 x dx = > > > > > > > : 1 para a > 6

a 1 x3 72 3 0

a3 216

para 0 a 6

Ejercicio 2

Variables aleatorias: Una gran simplicaci on!

22

Con las variables aleatorias hemos logrado una enorme simplicaci on (S, B, P ()) (RX , FX (x))
X ():S RX

o mejor. . .

(RX , fX (x)) (S, B, P ())

X ():S RX

Variables aleatorias: Una gran simplicaci on!

23

Ejemplo: (Tasa variaci on nd. NASDAQ) Pasamos de modelo tipo (S, B, P ())

S (result 1: a comprar a n acciones A a precio. . . cuando ayer. . . ), B(que ocurra resultado 1 y 2 pero no 3, etc.) P ()
(la probabilidad de que ocurra 1 y 2 pero no 3 es . . . )

(a trav es mercado de valores) a otro m as sencillo: Tasa de Variaci on NASDAQ se a distribuye Normal .

(RX , fX (x; )), (S, B, P ())


donde RX = (, ), = (, 2 ) y

X ():S RX

fX (x; ) =
a en

1 2 2

) exp (x2 2

realidad su distribuci on es m as parecida a una t-Student

De (S, B, P ()) a un modelo de probabilidad: historia en s mbolos

24

S RX (B, P ()) {fX (x; ) , } Llamamos Soporte al conjunto RX de posibles valores de la v.a. Llamamos Modelo de Probabilidad a la familia o colecci on de funciones de densidad = {fX (x; ) , , x RX } Llamamos Espacio Param etrico al conjunto de posibles valores de los par ametros de las funciones de densidad.

Modelado emp rico

25

1. Postulamos a priori una familia de densidades como mecanismo estoc astico subyacente. a) debemos elegir aquella familia m as adecuada a los datos histograma sugiere funcs. dens. fX (x; ) 0.8 datos pueden restringir espacio param etrico, el soporte, RX RX (si porcentaje, soporte [0, 1]) 2. Elegimos una familia, no una funci on en particular = {fX (x; ) , , x RX } los par ametros son desconocidos. 3. Inferencia estad stica valores de los par ametros siguientes. Tema 4 y

4. Variables aleatorias bivariantes

Sean X e Y denidas sobre

Vector aleatorio

26

(S1 , B1 , P1 ())

(S2 , B2 , P2 ())

Entonces, si denimos el conjunto de pares de sucesos: (s, z ) B(2) B1 B2 ;

la funci on Z (, ) : B(2) R2 , (s, z ) (X (s), Y (z )) es un vector aleatorio (una v.a respecto a B(2) ); con soporte conjunto: 2 RXY RX RY .

Vector aleatorio: funci on de distribuci on

27

El vector aleatorio (v.a. bivariante) Z (, ) = X (), Y () tiene funci on de distribuci ona : FXY (, ) : R2 XY [0, 1] FXY (x, y ) P(2) (s, z ) : donde X (s) x y Y (z ) y

a funci on

de distribuci on conjunta de X e Y

Variables discretas: Funci on de cuant a conjunta

28

En el caso univariante: P X (x) P (s; tales que X (s) = x)

En el caso bivariante: PXY (x, y ) P(2) (s, z ); tales que X (s) = x, Y (z ) = y

Funci on de cuant a: ejemplo

29

Lanzamiento de dos monedas: S1 = { , } S2 = { , } y con P ( ) = P ( ) = 1/2

Denamos las v.a.: sale cara, sale cruz: X( , ) = X( , ) = X( , ) = 1 Y( , ) = Y( , ) = Y( , ) = 1 X( , ) = 0 Y( , ) = 0

Las funciones de cuant a univariantes son: x 0 1 y P X (x) 0.25 0.75 P Y (y )

0 0.25

1 0.75

Funci on de cuant a conjunta: ejemplo

30

Sucesos que verican: (X = x, Y = y ); 2 donde (x, y ) RXY RX RY (X (X (X (X = 0, = 0, = 1, = 1, Y Y Y Y = 0) = 1) {( , )} = 0) {( , )} = 1) {( , ), ( , )} PXY (0, 0) = 0, PXY (0, 1) = 0.25, PXY (1, 0) = 0.25, PXY (1, 1) = 0.50

Es decir, funci on de cuant a conjunta: y \x 0 1 0 0 0.25 1 0.25 0.50

Funci on de cuant a conjunta reeja la dependencia (o independencia) entre las v.a.

Funci on de cuant a: Bivariante

31

0.50

0.25 1 0 0 1 x

0.17

Funci on de distribuci on y funci on de cuant a conjuntas

32

PXY (x, y ) X= 0 X= 1

Y =0 0/4 1/4

Y =1 1/4 2/4

FXY (a, b) = P (X a, Y b)

FXY (1.5, 0.5) = P (X 1.5, Y 0.5) = PXY (0, 0) + PXY (1, 0) = 0 + 1/4 = 1/4
calcular otros puntos (por ejemplo (0,0), (1,1), (-1,1.5),. . . )

Funci on de distribuci on: Bivariante

33

( 1 , 1 . 5) 1 0 0 y 1 (1 .5 , 0 .5)
x

0.18

Ejercicio 3. Sean X e Y variables aleatorias discretas con funci on de cuant a conjunta PXY (x, y ) = c(2x + y ) donde RX = {0, 1, 2} e RY = {0, 1, 2, 3} (a) Encuentre el valor de la constante c Soluci on : y \x 0 1 2 3 0 c0 c1 c2 c3 1 c2 c3 c4 c5 2 c4 c5 c6 c7

la suma de las probabilidades de todas las casilla debe ser 1; por tanto 42c = 1; es 1 decir c = 42 . As pues, la funci on de cuant a conjunta es: y \x 0 1 2 3 0 0/42 1/42 2/42 3/42 1 2/42 3/42 4/42 5/42 2 4/42 5/42 6/42 7/42 Ejercicio 3

(b) Calcule PXY (2, 1) Soluci on: Corresponde a la casilla u ltima de la segunda la, es decir, 5/42. Ejercicio 3 (c) Calcule P (X 1, Y 2) Soluci on: Los casos que cumplen la condici on son y \x 0 1 2 3 cuya probabilidad suma 24/42. (d) Calcule P (X + Y = 1) Soluci on : y \x 0 1 2 3 cuya probabilidad suma 3/42. 0 0/42 1/42 2/42 3/42 1 2/42 3/42 4/42 5/42 2 4/42 5/42 6/42 7/42 Ejercicio 3 0 0/42 1/42 2/42 3/42 1 2/42 3/42 4/42 5/42 2 4/42 5/42 6/42 7/42 Ejercicio 3

(e) Calcule P (X + Y 3) Soluci on : y \x 0 1 2 3 cuya probabilidad suma 24/42. 0 0/42 1/42 2/42 3/42 1 2/42 3/42 4/42 5/42 2 4/42 5/42 6/42 7/42 Ejercicio 3

Variables continuas: Funci on de densidad conjunta

34

Funci on de densidad conjunta, fXY (x, y ), de X e Y es la funci on cont nua y positiva que verica P (X a, Y b) FXY (a, b) =
(x,y )R2 XY xa y b 2 para todo (a, b) RXY .

fXY (x, y ) dxdy

Ya que fXY (x, y ) continua, tenemos FXY (x, y ) = fXY (x, y ) . xy

Funci on de densidad conjunta

35

fXY (x, y )
0.5 0

Y X

0.20

Ejercicio 4. La funci on de distribuci on conjunta de una exponencial bivariante es: FXY (x, y ) = 1 ex ey + exy , Calcule su funci on de densidad. Soluci on : fXY (x, y ) = d2 ` y e + eyx ex + 1 dxdy d ` x eyx = eyx . = e dy Ejercicio 4 x 0, y 0,

Ejercicio 5. (a) Dibuje la siguiente funci on de densidad conjunta uniforme ( k si 0 < x < 4 y 0 < y < 2 fXY (x, y ) = 0 en el resto de casos (b) Cuanto vale la altura k Soluci on : Z
0 2

Z
0

fXY (x, y ) dx dy =1 Z
0 2

4k dy =1 8k =1 k =1/8 Ejercicio 5

Ejercicio 6. Sea la siguiente funci on de densidad ( k si 0 < x < 2y < 4 fXY (x, y ) = 0 en el resto de casos (a) Calcule el valor de k Soluci on: Hay dos posibilidades para resolverlo

2
0
1. Z 1=
0 0 4 0 2

x=

2y x/2

y=

4
Z
2y

x
Z
2

k dx dy = 2k
0 2

y dy = 4 k R4
0

1 k= 4 1 k= 4 Ejercicio 6

2. Z 1=
x/2

k dy dx =

x dx

=4k

(b) Calcule P (Y > 1) . Soluci on: De nuevo hay dos posibilidades

y 2
k

x=

2y x/2

y=

1 0 2
1 dx dy = 4 Z
2 x/2

4
R2
1

x
= 3 4
x 2

1. Z P (Y > 1) =
1 0 2

2y

y dy 2 R4
2

2. Z P (Y > 1) =
0 1 2

1 dy dx + 4

Z
2

1 dy dx = 4

2 4

dx

1 3 = 2 4

Ejercicio 6 Soluci on num erica con octave

Ejercicio 7. Sea la funci on de densidad continua, ( k si 0 < x < y < 2 . fXY (x, y ) = 0 resto Indique c omo calcular a la siguiente probabilidad: P (X + Y 1). Soluci on: La regi on que cumple la condici on es
y 2
x

y x + y = 1

1/2

Por tanto: P (X + Y 1) =

R 1/2 R 1x
0 x

k dy dx

Ejercicio 7

Calculo num erico de la probabilidad con octave

Ejercicio 8. Sean X e Y variables aleatorias con funci on de densidad conjunta fXY (x, y ) para alg un valor concreto de k: k si 1 < x < 1; 1 < y < 1 fXY (x, y ) = 0 resto de casos Determine la probabilidad de los siguientes eventos: (a) 2X Y > 0. Soluci on: Puesto que el area del soporte es 4, y la funci on de densidad es uniforme; para que fXY (x, y ) sea una funci on de densidad k (la altura) debe ser igual a 1/4. El suceso 2X Y > 0 divide el espacio de sucesos (el cuadrado de la gura) en dos areas iguales; y como la distribuci on es uniforme, cada una de ellas tiene probabilidad igual a 1/2
1
y= 2x

-1

-1

Ejercicio 8

(b)

X2 + Y2 < 1 Pista. X 2 + Y 2 < r dene un c rculo de radio r centrado en (0, 0), y con area igual a r 2 . Soluci on: La probabilidad de X 2 + Y 2 < 1 es igual al volumen del cilindro con base de area 12 = y altura 1/4, es decir, ` P X 2 + Y 2 < 1 = base altura = 4
1

-1

Area =

-1

Ejercicio 8

(c)

X 2 + Y 2 < 1, y simult aneamente 2X Y > 0. Soluci on: Por el mismo argumento, la probabilidad del suceso conjunto X 2 + Y 2 < 1 y 2X Y > 0 es la mitad de la probabilidad del suceso X 2 + Y 2 < 1; por lo tanto . igual a 8
1
y= 2x

-1

Area =

-1

Ejercicio 8 Calculo num erico de la probabilidad con octave

Ejercicio 9. Sean X e Y variables aleatorias con distribuci on conjunta fXY (x, y ). La integral sobre su soporte conjunto RXY es: Z 1 Z x2 +1 fXY (x, y ) dy dx.
1 x2

Dibuje el soporte RXY . Soluci on :

-1

Ejercicio 9

Propiedades de las leyes de probabilidad bivariantes

36

caso continuo (funci on de densidad) fb1. fXY (x, y ) fb2.


R2 XY

0 para todo (x, y ) R2 XY = 0 en el resto de casos

fXY (x, y ) dydx = 1 fXY (x, y ) dydx,


(x,y )R2 XY xa; y b

fb3. FXY (a, b) =

caso discreto (funci on de cuant a) fb1. PXY (x, y ) fb2.


2 0 para todo (x, y ) RXY

= 0 en el resto de casos PXY (x, y ) = 1


(x,y )R2 XY xa; y b

(x,y )R2 XY

fb3. FXY (a, b) =

PXY (x, y )

Ejercicio 10. Sean X e Y variables aleatorias con distribuci on conjunta fXY (x, y ). La integral sobre su soporte conjunto RXY es: Z 1 Z 1 fXY (x, y ) dy dx = 1.
1 x2

Dibuje el soporte RXY . Soluci on :


1 > y > x2

-1

Ejercicio 10

5. Distribuciones marginales

Distribuciones conjuntas y marginales: ejemplo

37

Sea la siguiente funci on de cuant a conjunta: y \x 0 2 0 0.2 0.1 1 0.2 0.1 2 0.2 0.2

Las funciones de cuant a marginales son: x P X (x) 0 0.3 1 0.3 2 0.4 y P Y (y ) 0 0.6 2 0.4

N otese que funciones de cuant a marginales son leyes de probabilidad (suman 1)!

P Y (y )

Funci on de cuant a marginal

38

0.6

0.4

y
0.2 2

0 0 1

Distribuciones conjuntas y marginales

39

PXY (x, y ) contiene informaci on sobre la relaci on entre X e Y .

Puesto que son posibles muchos tipos de relaci on entre X e Y ; en general, no es posible partiendo de P X (x) y P Y (y ) obtener PXY (x, y )

(ser a necesario decir, adem as, como se relacionan X e Y )

(lo mismo se puede decir en el caso cont nuo)

Distribuciones conjuntas y marginales

40

Siempre es posible deducir P X (x) y P Y (y ) a partir de PXY (x, y ): Caso discreto: Funci on de cuant a marginal de X P X (xi ) =
yj RY

PXY (xi , yj )

Funci on de cuant a marginal de Y P Y (yi ) =


xi RX

PXY (xi , y )

Caso continuo: fX (x) =


y RY

fXY (x, y ) dy

Ejercicio 11. [Continuaci on del Ejercicio 3 en la p agina60] Calcule las funciones marginales de la siguiente funci on de cuant a conjunta y \x 0 1 2 3 Soluci on : y \x 0 1 2 3 P X (x) 0 0/42 1/42 2/42 3/42 6/42 1 2/42 3/42 4/42 5/42 14/42 2 4/42 5/42 6/42 7/42 22/42 P Y (y ) 6/42 9/42 12/42 15/42 Ejercicio 11 0 0/42 1/42 2/42 3/42 1 2/42 3/42 4/42 5/42 2 4/42 5/42 6/42 7/42

Ejercicio 12. [Continuaci on del Ejercicio 6 en la p agina67] Calcule las funciones de densidad marginales fX (x) y fY (y ). Calculo num erico con octave Soluci on : Z fX (x) =
x/2 2

fXY (x, y ) dy = Z
2y

2 4 y 2

x 2

para 0 < x < 4 para 0 < y < 2 Ejercicio 12

fY (y ) =
0

fXY (x, y ) dx =

Distribuciones conjuntas y marginales

41

fY (y ) fX (x) fXY (x, y )


0.5 0

Y X

Leyes de prob. marginales pierden la informaci on de otras variables Son leyes de probabilidad (integran 1)! Calculo num erico con octave

6. Distribuciones condicionadas

6.1. Caso discreto

Funciones de cuant a condicionadas

42

P ( A | B)

P (A B ) P (B ) P X (x) ,

siempre que P (B ) > 0.

PXY (x, y ) PXY (b, y ) P X (b)

P Y|X ( y | b) para un valor b jo, y P X (b) = 0 P Y|X ( y | b) es funci on de y .

Funciones de cuant a condicionales son leyes de probabilidad (suman 1)!

Probabilidad conjunta, marginal y condicionada

43

y \x 0 1 2 P X (x)

1 0.20 0.10 0.01

2 0.10 0.25 0.06

3 0.15 0.05 0.08

P Y (y )

X = 1 : (18 35) Y = 0 : pobre 1. Quien soy? 2. Que edad tengo?

X = 2 : (36 55) X = 3 : (56 70) Y = 1 : renta intermedia Y = 2 : rico

3. Que edad tengo si conduc a un Porsche?

Funci on de cuant a condicionada

44

0.5 0.6

0.4

0.25

y
0.2 2

0 0 1

P X|Y ( x | 2)

Ejercicio 13. Sea (X, Y ) una variable aleatoria bidimensional discreta, donde X e Y pueden tomar los valores 0,1,2,3. La variable aleatoria Y hace referencia a los cambios en los ingresos, mientras que X hace menci on a los cambios en los gastos en publicidad. Denimos las variables de la siguiente manera: Y = 0 si las ventas caen, Y = 1 si las ventas se mantienen, Y = 2 si las ventas suben, pero menos de un 10 %, Y = 3 si suben m as de un 10 %. De manera an aloga denimos X . La ley de probabilidad conjunta viene dada por PXY (x, y ) = k(3 x + y ). (a) Halle el valor de k para que PXY (x, y ) sea una verdadera ley de probabilidad. PP Soluci on: Tenemos que k(3 x + y ) = 48k, por lo que k = 1/48. Ejercicio 13 (b) Para el correcto valor de k, represente en una tabla de doble entrada la funci on de cuant a de la variable aleatoria bidimensional. Soluci on : Y \X 0 1 2 3 0 3/48 4/48 5/48 6/48 1 2/48 3/48 4/48 5/48 2 1/48 2/48 3/48 4/48 3 0 1/48 2/48 3/48 Ejercicio 13

que como se puede comprobar suma 1.

(c) Calcule las leyes de probabilidad marginales de X e Y . Seg un esta informaci on, es m as probable que las ventas bajen, se mantengan o suban? Soluci on: Las leyes de probabilidad marginales de X e Y vienen dadas por: P X ( x) X
yj RY

PXY (x, yj ) =

3 X yj =0 3 X xj =0

PXY (x, yj ) para todo x RX

P Y (y ) es decir,

X
xj RX

PXY (xj , y ) =

PXY (xj , y ) para todo y RY

X P X (x) Y P Y (y )

0 18/48 0 6/48

1 14/48 1 10/48

2 10/48 2 14/48

3 6/48 3 18/48

Por lo tanto, P (caer) = P Y (0) = 0.125 P (mantener) = P Y (1) = 0.208 P (subir) = P Y (2) + P Y (3) = 0.667; Es m as probable que suban. Esta probabilidad est a calculada sobre la base de las leyes de probabilidad marginales de Y . Esto es, no tenemos ninguna informaci on

precisa acerca de la realizaci on de la variable aleatoria X . Dicha informaci on nos podr a ayudar a ser m as precisos en el c alculo de estas probabilidades. Ejercicio 13

(d) Halle las leyes de probabilidad condicionadas de Y con respecto a los diferentes valores que pueda tomar X . Var an entre ellas y con respecto a la ley de probabilidad marginal de Y ? Que signica la respuesta a la pregunta anterior? Si X = 0, cambiar a la respuesta cualitativa del apartado anterior? Y si X = 2?

Soluci on: Las leyes de probabilidad condicionadas est an resumidas en la siguiente tabla. N otese que tenemos 4 posibles variables aleatorias, una para cada posible valor de X . Para cada una de estas v.a. tenemos una ley de probabilidad condicionada.
X =0 P Y| X( y | 0) Y =0 Y =1 Y =2 Y =3 X =1 P Y| X( y | 1) X =2 P Y| X( y | 2) X =3 P Y| X( y | 3)

3/18 4/18 5/18 6/18

2/14 3/14 4/14 5/14

1/10 2/10 3/10 4/10

0 1/6 2/6 3/6

Usando la ley de probabilidades apropiada en cada caso: Si X = 0: P ( caer | X = 0) = P Y|X ( 0 | 0) = 0.167 P ( mantener | X = 0) = P Y|X ( 1 | 0) = 0.222 P ( subir | X = 0) = P Y|X ( 2 | 0) + P Y|X ( 3 | 0) = 0.611 Si X = 1: P ( caer | X = 1) = P Y|X ( 0 | 1) = 0.143 P ( mantener | X = 1) = P Y|X ( 1 | 1) = 0.214 P ( subir | X = 1) = P Y|X ( 2 | 1) + P Y|X ( 3 | 1) = 0.643 Si X = 2: P ( caer | X = 2) = P Y|X ( 0 | 2) = 0.1 P ( mantener | X = 2) = P Y|X ( 1 | 2) = 0.2

P ( subir | X = 2) = P Y|X ( 2 | 2) + P Y|X ( 3 | 1) = 0.7 Si X = 3: P ( caer | X = 3) = P Y|X ( 0 | 3) = 0 P ( mantener | X = 3) = P Y|X ( 1 | 3) = 0.167 P ( subir | X = 3) = P Y|X ( 2 | 3) + P Y|X ( 3 | 3) = 0.833 Las probabilidades han cambiado. Esto nos indica que las variables son dependientes. Nueva informaci on sobre X hace cambiar las leyes de probabilidad sobre los ingresos. Ejercicio 13

(e) Seg un el apartado anterior, para qu e valor de X es m as probable que las ventas aumenten m as de un 10 %? Para qu e valor de X es m as probable que las ventas caigan? Nos da esto alguna indicaci on del posible signo de la relaci on entre las dos variables? Soluci on: La probabilidad de que los ingresos suban aumenta a medida que el gasto en publicidad es mayor P ( subir | X = 0) < P ( subir | X = 1) < P ( subir | X = 2) < P ( subir | X = 3) . Mientras que la probabilidad de que los ingresos caigan crece cuanto menor son los gastos en publicidad. P ( caer | X = 0) > P ( caer | X = 1) > P ( caer | X = 2) > P ( caer | X = 3) . As pues, podemos sospechar que la relaci on entre X e Y es positiva; es decir, cuando mayor es X , mayor es la probabilidad de tener valores altos en Y y viceversa. Ejercicio 13

(f) Denamos Z = Y X . Cu al es la probabilidad de que Z > 0? Y de que Z = 0? Y de Z < 0? Soluci on: Reescribimos las probabilidades de Z en t erminos de una relaci on entre X e Y . De este modo podemos usar la informaci on sumistrada por la ley de probabilidad conjunta, que s es conocida. As P (Z > 0) = P (Y > X ) = PXY (0, 1) + PXY (0, 2) + + PXY (2, 3) = 28/48 = 0.583 N otese que la diagonal principal muestra el caso en que X = y , o sea Z = 0. Lo que est a por debajo de la diagonal principal son los casos en que Y > X (Z > 0) y por encima de la diagonal principal los casos en que Y < X (Z < 0). De manera an aloga: P (Z = 0) = P (Y = X ) = 12/48 = 0.250;

P (Z < 0) = P (Y < X ) = 8/48 = 0.167 Ejercicio 13

(g) C omo cambian las respuestas del apartado anterior si X = 0, o X = 1, oX =2 o X = 3 (use la informaci on de las probabilidades condicionadas?) Soluci on: Tenemos que reescribir estas probabilidades en t erminos de las leyes de probabilidad condicionadas: Si X = 0: P ( Z > 0 | X = 0) = P ( Y X > 0 | X = 0) = P Y|X ( 1 | 0) + P Y|X ( 2 | 0) + P Y|X ( 3 | 0) = 15/18 = 0.833 P ( Z = 0 | X = 0) = 3/18 = 167 De manera an aloga lo har amos para los dem as posibles valores de X .

A lo largo de todo el problema la clave ha estado en identicar correctamente la variable aleatoria (marginal, condicionada o conjunta) sobre la que nos estamos preguntando y usar en cada momento la ley de probabilidades asociada a ella. Ejercicio 13

6.2. Caso continuo

La probalidad del suceso A condicionada al suceso B se dene como P ( A | B) P (A B ) , P (B ) siempre que P (B ) > 0.

El siguiente ejercicio puede resolverse aplicando lo anterior ya que la probabilidad de la condici on es mayor que cero:

Ejercicio 14. Sea la funci on de densidad bivariante ( 2(x + y ) si 0 < y < x < 1 fXY (x, y ) = . 0 en el resto del plano ` Calcule P 0.2 X 0.4 Y < 0.5 . Soluci on: Por una parte ` P (0.2 X 0.4) P (Y < 0.5) P 0 .2 X 0 .4 Y < 0 . 5 = . P (Y < 0.5) Por otra Z
1

fY (y ) =
y

2(x + y ) dx = 1 + 2y 3y 2 ;
0.5 0

por tanto Z P (Y < 0.5) = y 1 + 2y 3y 2 dy = 0.625, Z P (0.2 X 0.4) P (Y < 0.5) =


0.2 0 0.4

2(x + y ) dydx = 0.056.

As pues, ` 0.056 P 0 .2 X 0 .4 Y < 0 . 5 = = 0.0896. 0.625 Ejercicio 14

Sin embargo, si la pregunta al ejercicio anterior hubiera sido calcular P ( 0.2 X 0.4 | Y = 0.5) ; on cont nua y por tanto P (Y = 0.5) = 0, entonces la y puesto que Y es una funci estrategia anterior no hubiera sido posible ya que la probabilidad de la condici on es cero.

Distribuciones de probabilidad condicionadas: Caso continuo

45

Funci on de densidad condicionada f Y|X ( y | a) Funci on de distribuci on condicionada F Y|X ( b | a)


(a,y )R2 XY y<b

fXY (a, y ) fX (a) fXY (a, y ) dy fX (a)


(a,y )R2 XY y<b

f Y|X ( y | a) dy =

para un valor a jo, y fX (a) = 0

Funciones de densidad condicionales son leyes de probab. (integran 1)!

Distribuciones condicionadas

46

fY (b)

f X|Y ( x | b) fY (y ) fX (x) fXY (x, y )

Y
0.44

Ejercicio 15. Sean X e Y con funci on de densidad uniforme igual a 2 en el tri angulo 0 x y k, es decir, ( 2 para 0 x y k fXY (x, y ) = 0 en el resto Calcule lo siguiente:

(a) El valor de k para que fXY (x, y ) sea un funci on de densidad Soluci on: El soporte es
y

Por ser la distribuci on uniforme sobre el soporte triangular, la doble integral coincide con el volumen del quesito sobre dicho soporte, que debe ser igual a uno. El volumen es igual en este caso (distribuci on uniforme) a la supercie del tri angulo multiplicada por la altura de la funci on, que es 2. Por tanto superf icie altura = superf icie 2 = 1 de donde sabemos que la supercie es 1/2. Puesto que la supercie de un tri angulo es base por altura dividido por dos. Y en este caso base y altura son iguales a k, necesariamente k = 1.

Otra forma de calcular m as general es Z k Z Z kZ k 2dydx = [2y ]k = x


0 x 0

2k 2xdx =
0

k = 2kx x2 0 = 2k2 k2 = k2 = 1 de donde k = 1. Ejercicio 15

(b) Las funciones marginales de X e Y Soluci on : Z fX (x) =


1 x

2dy = 2(1 x); para x [0, 1] Z y 2dx = 2y ; para y [0, 1] fY (y ) =


0

Ejercicio 15 (c) Las funciones de densidad condicionadas f Y|X ( y | a) y f X|Y ( x | b) Soluci on : fXY (x, b) 1 = ; para 0 x b fY (b) b fXY (a, y ) 1 f Y|X ( y | a) = = ; para a y 1 fX (a) 1a f X|Y ( x | b) = Ejercicio 15

` (d) P Y >

1 2

X = a para cualquier valor factible a RX 8 > 1 > > > <


1 2

Soluci on : si a 1 Y> X=a = 1 1 R R > 2 > > f Y|X ( y | a) dy = > :


1/2

1/2

1 dy 1a

0.5 1a

si a <

1 2

Ejercicio 15 ` (e) P X > Soluci on : 8 > 0 > > > < si b


1 2 1 2

Y = b para cualquier valor factible b RY

1 X> Y = b = b b > R R 2 > > f X|Y ( x | b) dx = > :


1/2

1/2

1 dx b

=1

1 2b

si b >

1 2

Ejercicio 15

` (f) P Y > X 2 , X > Y 2 Soluci on: El area a integrar es


y 1 y = x2 x = y2


1 0 y

es decir ` P Y > X2, X > Y2 = =


0

Z Z

Z 2dxdy =
0

y2 1

h iy 2x 2 dy =
y

h 2y 3 i1 1 2y 2y 2 dy = y 2 = 3 0 3 Ejercicio 15

(g) Sin realizar m as c alculos, podr a decir cual es la probabilidad de ` P Y > X2, X > Y2, Y > 1 X (con decir en que proporci on es mayor, menor (o igual) a la anterior probabilidad es suciente) Y si fXY (x, y ) no fuera uniforme? area a integrar es Soluci on: El
y 1 y = x2 x = y2

= 1

que es exactamente la mitad del area anterior, por lo tanto, la probabilidad es exactamente la mitad de la calculada en el apartado anterior. Si la funci on de densidad no fuera uniforme, la mitad del area no implicar a necesariamente mitad de la probabilidad, ya que habr a regiones con mayor densidad de probabilidad que otras. Ejercicio 15 Continuaci on en el Ejercicio ?? en la p agina??.

Ejercicio 16. Sea la siguiente funci on de densidad ( k(x + y ) si 0 < y < x < 1 fXY (x, y ) = . 0 en el resto de casos C omo calcular a la probabilidad condicionada P ( Y > 0.5 | X (0.25, 0.75))? (NO lo calcule, pero indique correctamente los ordenes de integraci on) Soluci on: Hay varias posibles respuestas dependiendo del orden en que se integran las variables. Aqui presentamos dos posibles respuestas

0.5


0
.25

1
.75

1.

La probabilidad condicionada P ( Y > 0.5 | X (0.25, 0.75)) es R 0.75 R 0.75 R 0.75 R 0.75 fXY (x, y ) dx dy fXY (x, y ) dx dy y y 0.5 0.5 = R 0.75 R x R 0.75 0.25 0 fXY (x, y ) dy dx 0.25 fX (x) dx Rx donde la funci on de densidad marginal de X es fX (x) = 0 fXY (x, y ) dy. La probabilidad condicionada: P ( Y > 0.5 | X (0.25, 0.75)) es R 0.75 R x R 0.75 R x .5 fXY (x, y ) dy dx 0.5 fXY (x, y ) dy dx 0.5 0.5 = R R R0 0.75 x 0.75 0.25 0 fXY (x, y ) dy dx 0.25 fX (x) dx Rx donde la funci on de densidad marginal de X es fX (x) = 0 fXY (x, y ) dy. Ejercicio 16

2.

7. Independencia

Independencia

47

P ( A | B) = P (A) ; P (B ) = 0 P ( B | A) = P (B ) ; P (A) = 0 Caso discreto:

P (A B ) = P (A) P (B )

P Y|X ( y | x) = P Y (y ) para cualquier x; PXY (x, y ) = P X (x) P Y (y ) Caso continuo: f Y|X ( Y | x) = fY (y ) para cualquier x; fXY (x, y ) = fX (x) fY (y )

convertir test en ejercicio Test. Conteste a la siguiente cuesti on. 1. Sean X e Y con fXY (x, y ) = exy donde x e y son mayores que cero (exponencial bivariante). (a) X e Y son independientes Soluci on:Puesto que Z fX (x) =
0

(b) X e Y NO son independientes exy dy = ex exy dx = ey .

y Z fY (y ) =
0

Entonces fX (x) fY (y ) = ex ey = exy = fXY (x, y ) Fin 1

Ejercicio 17. Sean dos variables aleatorias discretas, (X ,Y ), que reejan la siguiente informaci on: X = 1 si se da un shock positivo de demanda en la econom a y X = 1 si el shock es negativo; Y = 1 si el salario real sube, Y = 1 si el salario real baja e Y = 0 si se mantiene. La rigidez salarial implica que los salarios reales no se ven afectados por shocks de demanda. Seg un la siguiente tabla de probabilidades conjuntas, podr amos armar que los salarios reales son r gidos en esta econom a? PXY (x, y ) Y -1 0 1 X -1 1 1/18 2/18 3/18 6/18 2/18 4/18

Soluci on: Salarios r gidos signica que X e Y son independientes, es decir, que PXY (x, y ) = P X (x) P Y (y ) para todo x e y . Puesto que las funciones de a marginales son: 8cuant > ( 3/18 si Y = 1 > < 6/18 si X = 1 P X ( x) = , y P Y (y ) = 9/18 si Y = 0 ; cuyo producto coincide > 12/18 si X = 1 > :6/18 si Y = 1 con la funci on de cuant a conjunta; los salarios reales son r gidos Ejercicio 17

8. Momentos conjuntos

Momentos conjuntos: caso discreto

48

E (g (X, Y )) =

2 (x,y )RXY

g (x, y ) PXY (x, y )

Respecto al origen: arm = E XY (X r Y m ) = para r, m = 0, 1, 2, . . . Respecto al valor esperado: rm = E XY ([X E X (X )]r [Y E Y (Y )]m ) =
2 (x,y )RXY 2 (x,y )RXY

xr y m PXY (x, y )

(x X )r (y Y )m PXY (x, y )

para r, m = 0, 1, 2, . . .

Momentos conjuntos: caso cont nuo

49

E (g (X, Y )) =
(x,y )R2 XY

g (x, y ) f XY (x, y ; ) dx dy

Respecto al origen: arm = E XY (X r Y m ) =


R2 XY

xr y m fXY (x, y ) dydx

para r, m = 0, 1, 2, . . . Respecto al valor esperado: rm = E XY ([X E X (X )]r [Y E Y (Y )]m ) =


R2 XY

(x X )r (y Y )m fXY (x, y ) dydx

para r, m = 0, 1, 2, . . .

Ejercicio 18. Sean X e Y variables aleatorias con distribuci on conjunta fXY (x, y ); y cuyo soporte conjunto RXY aparece en la siguiente gura:
y
y = x 1 y

1 0

Indique c omo calcular a E (X ) (indique correctamente los l mites de integraci on). Soluci on : Z
0 1

y +1

x fXY (x, y ) dx dy.


y

o tambi en Z
0 1

Z
0

Z x fXY (x, y ) dy dx +
1

x fXY (x, y ) dy dx.


x1

Ejercicio 18

8.1. Propiedades de los momentos bivariantes

Momentos te oricos bivariantes: propiedades

50

E XY (X + Y ) = E X (X ) + E Y (Y ) E XY (X Y ) = E X (X ) E Y (Y ) E X (aX ) = a E X (X )

y si X e Y son independientes E XY (X Y ) = E X (X ) E Y (Y )

Proposici on 8.1. E XY (g (X )) = E X (g (X )) donde fXY (x, y ) = 0 para (x, y ) R2 XY y fXY (x, y ) = 0 en el resto de casos. Demostraci on. E XY (g (X )) =
(x,y )R2 XY

g (x)fXY (x, y ) dydx =


2 (x,y )RXY

g (x)f Y|X ( Y | x) fX (x) dydx 3

2 Z =
xRX

6 6 g (x)fX (x) 6 6 4

(a,y )R2 XY aRX

7 7 f Y|X ( y | a) dy 7 7 dx 5

Z =
xRX

g (x)fX (x) dx = E X (g (X ))

Por esto cuando escribimos E (X ) nos referimos indistitamente tanto a E X (X ) como a E XY (X )

Ejercicio 19. Teniendo en cuenta lo anterior: (a) demuestre la primera y segunda propiedad de la trasparencia anterior . Soluci on : E (X + Y ) =
(x,y )R2 XY

(x + y )fXY (x, y ) dydx xfXY (x, y ) dydx +


2 (x,y )RXY 2 (x,y )RXY

yfXY (x, y ) dydx

= E (X ) + E (Y ) [Repita la demostraci on para el caso discreto (soluci on en Novales, 1997, Teorema 7.4)] La demostraci on de E XY (X Y ) = E X (X ) E Y (Y ) es similar. Ejercicio 19 (b) demuestre la tercera propiedad de la trasparencia anterior . Soluci on: No hay mas que sacar la constante a de la integral (o el sumatorio) y aplicar la denici on de E (X ). Ejercicio 19

(c) Demuestre tambi en la cuarta propiedad Soluci on: Basta con recordar que indenpendencia implica f Y|X ( Y | x) = fY (y )

y operar oportunamente; es decir E (X Y ) =


(x,y )R2 XY

x y fXY (x, y ) dydx x y f Y|X ( Y | x) fX (x) dydx


(x,y )R2 XY

= 2 x4
xRX

Z = Z =
xRX

Z
(x,y )R2 XY xRX dado

3 yf Y|X ( Y | x) dy 5 fX (x) dx # yfY (y ) dy fX (x) dx

por ser xfX (x) cte. r

"Z x
y RY

por in

Z =
xRX

x [E (Y )] fX (x) dx Z

por denici on de esp

= E (Y )
xRX

xfX (x) dx

por ser E (Y ) un n ume

= E (Y ) E (X ) (para el caso discreto la demostraci on es similar) Ejercicio 19

2 Ejercicio 20. Sean X e Y con funci on de densidad fXY (x, y ) y como soporte RXY el tri angulo 0 1 x y 1. Cu al (o cuales) de las siguientes expresiones es (o son) correcta (correctas)? R1 R1 1. E (X ) = 0 x 1x fXY (x, y ) dydx R1 R1 2. E (X ) = 0 x 0 fXY (x, y ) dydx R1 Ry 3. E (X ) = 0 x 1x fXY (x, y ) dydx R1R1 4. 1 = 0 0 fXY (x, y ) dydx

Soluci on :
y 1


y = 1 x

S olo la primera.

Ejercicio 20

Covarianza

51

El momento conjunto m as empleado es la covarianza: Cov (X, Y ) 11 = E XY ([X E X (X )][Y E Y (Y )]) XY Propiedades de la covarianza: C1. Cov (X, Y ) = E (XY ) E (X ) E (Y ) C2. Cov (X, Y ) = Cov (Y, X ) C3. Cov (aX, Y ) = aCov (X, Y ) para a R C4. Cov (X + Y, Z ) = Cov (X, Z ) + Cov (Y, Z )

Empleando la covarianza podemos extender la propiedad V2 en la p agina277 a v.a. no independientes : V1. Var (aX1 + bX2 ) = a2 Var (X1 ) + b2 Var (X2 ) + 2ab Cov (X, Y )

Ejercicio 21. Calcular Cov (X, Y ) dada la siguiente funci on de cuant a conjunta y \x 0 2 P X ( x) 0 0.2 0.1 0.3 1 0 .2 0 .1 0 .3 2 0 .2 0 .2 0 .4 P Y (y ) 0 .6 0 .4 1

Soluci on: Primero calculamos los momentos univariantes E (X ) =0 (0.3) + 1 (0.3) + 2 (0.4) = 1.1 E (Y ) =0 (0.6) + 2 (0.4) = 0.8 Var (X ) =[0 1.1]2 (0.3) + [1 1.1]2 (0.3) + [2 1.1]2 (0.4) = 0.69 Var (Y ) =[0 0.8]2 (0.6) + [2 0.8]2 (0.4) = 0.96 Ahora podemos calcular Cov (X, Y ): E (XY ) =(0)(0) (0.2) + (0)(2) (0.1) + (1)(0) (0.2)+ =(1)(2) (0.1) + (2)(0) (0.2) + (2)(2) (0.2) = 1 tenemos Cov (X, Y ) = E (XY ) E (X ) E (Y ) = 1 1.1 0.8 = 0.12 Ejercicio 21

Coeciente de correlaci on

52

Una debilidad de la covarianza es que depende de las unidades de medida. Para evitar este problema empleamos una versi on estandarizada: Corr (X, Y ) = XY = Cov (X, Y ) Var (X ) Var (Y ) = XY X Y

Propiedades del coef. correlaci on: 1. 1 Corr (X, Y ) 1 2. Corr (aX + b, cY + d) = Corr (X, Y ) , para (a, b, c, d) R4 , y (a b) > 0 3. Corr (X, Y ) = 1 si y s olo si Y = a + bX, (a, b) R2 .

Es una medida del grado de relaci on lineal entre X e Y Si X e Y son indep., = XY = 0 (pero ).

Ejercicio 22. Discuta la veracidad o falsedad de la siguiente armaci on: Si dos variables X , Y poseen un coeciente de correlaci on lineal muy pr oximo a cero, debemos concluir que pr acticamente no existe ninguna relaci on entre las dos variables. Soluci on: La conclusi on es falsa. La correlaci on lineal indica el grado de relaci on lineal entre dos variables. Si el coeciente de correlaci on lineal est a muy pr oximo a cero, esto quiere decir que la relaci on lineal entre las dos variables es pr acticamente inexistente. Pero esto no es obice para que pueda existir una fuerte relaci on (no lineal) entre las variables. Ejercicio 22

Ejercicio 23. (Consta de 5 apartados) Sea (X, Y ) una variable aleatoria bidimensional discreta, donde X e Y pueden tomar los valores 0, 1 o 2. La variable aleatoria X hace referencia al volumen de venta de bebidas on alcoh olicas en un chiringuito de playa en un d a cualquiera, mientras que Y hace menci al volumen de ventas de refrescos el mismo d a en el mismo establecimiento. Denimos las variables de la siguiente manera: X = 0 si se vende poco, X = 1 si el volumen de ventas es medio y X = 2 si se vende mucho. De manera an aloga denimos Y . La ley de probabilidad conjunta viene dada por la siguiente funci on de cuant a: HH Y 0 1 2 X HH H 0 0.12 0.16 0.12 1 0.08 0.12 0.16 2 0.04 0.08 0.12

(a) Calcule las leyes de probabilidad marginales de X e Y . Seg un esta informaci on, que situaci on es m as probable respecto a las ventas de alcohol? Soluci on :

HH Y X HH H
0 1 2 P Y (y )

0 0.12 0.08 0.04 0.24

1 0.16 0.12 0.08 0.36

2 0.12 0.16 0.12 0.40

P X ( x) 0.40 0.36 0.24

Por lo tanto, P (poco) = P X (0) = 0.40 P (medio) = P X (1) = 0.36 P (mucho) = P X (2) = 0.24 es decir, lo m as probable es vender pocas bebidas alcoh olicas.

Ejercicio 23

(b) Halle las leyes de probabilidad condicionadas de X con respecto a los diferentes valores que pueda tomar Y . Var an entre ellas y con respecto a la ley de probabilidad marginal de X ? Que signica la respuesta a la pregunta anterior? Si Y = 0, cambiar a la respuesta cualitativa del apartado anterior? Y si Y = 2? Soluci on: Las leyes de probabilidad condicionadas est an resumidas en las columnas de la siguiente tabla.
P X| Y ( x | 0) X =0 X =1 X =2 P X| Y ( x | 1) P X| Y ( x | 2)

0.500 0.333 0.167

0.44 0.33 0.22

0.300 0.400 0.300

Las probabilidades han cambiado. Esto nos indica que las variables son dependientes. Nueva informaci on sobre X hace cambiar las leyes de probabilidad sobre las ventas de bebidas alcoh olicas; no obstante, si la venta de refrescos ha sido baja (Y =0) sigue siendo m as probable que la venta de alcohol sea baja. Sin embargo, si la venta de refrescos ha sido elevada (Y =2) lo m as probable es que haya un volumen intermedio de ventas de bebidas alcoh olicas. Ejercicio 23

(c) Calcule la correlaci on entre la venta de refrescos y la de bebidas alcoh olicas. Interprete el resultado. Soluci on: Necesitamos calcular las desviaciones t picas y la covarianza de X e Y . ` Por una parte, E (X ) = 0.36 + 2 0.24 = 0.84 y por otra E X 2 = 0.36 + 22 0.24 = 1.32 Por tanto ` 2 2 Var (X ) = E X 2 E (X ) = 1.32 0.84 = 0.6144 ` En cuanto a Y , E (Y ) = 0.36 + 2 0.40 = 1.16 y E Y 2 = 0.36 + 22 0.40 = 1.96 As pues, ` 2 2 Var (Y ) = E Y 2 E (Y ) = 1.96 1.16 = 0.6144 La covarianza es Cov (X, Y ) = E (XY ) E (X ) E (Y ) = = 0.12 + 2 0.16 + 2 0.08 + 4 0.12 0.84 1.16 = 0.1056 Finalmente el coeciente de correlaci on es Cov (X, Y ) 0.1056 p Corr (X, Y ) = p = = 0.17188 0.6144 0.6144 Var (X ) Var (Y ) Existe una d ebil relaci on lineal de signo positivo entre las variables. Ejercicio 23

(d) El due no del chiringuito considera mal d a a aquel en el que las ventas bajas de un tipo de bebida, no son compensadas con ventas elevadas del otro tipo de bebida (es decir si X + Y < 2 ). C ual es la probabilidad de que se d e un mal d a? Soluci on: Los casos que generan un mal d a se pueden resumir en aquellos que verican X + Y < 2. As pues, P (X + Y < 2) = PXY (0, 0) + PXY (0, 1) + PXY (1, 0) = 0.36 Ejercicio 23 (e) C omo cambian las respuestas del apartado anterior si se sabe con anterioridad que Y = 0, que Y = 1, o que Y = 2 (use la informaci on de las probabilidades condicionadas?) Soluci on: Tenemos que reescribir estas probabilidades en t erminos de las leyes de probabilidad condicionadas: Si Y = 0: P ( X + Y < 2 | Y = 0) = P X|Y ( 0 | 0) + P X|Y ( 1 | 0) = 0.50 + 0.333 = 0.833 Si Y = 1: P ( X + Y < 2 | Y = 1) = P X|Y ( 0 | 1) = 0.44 Si Y = 2: P ( X + Y < 2 | Y = 2) = 0 Ejercicio 23

9. Momentos condicionados

Momentos condicionados: caso discreto

53

Esperanza condicionada: E Y|X ( Y | a) =


y RY

y P Y|X ( y | a)

Varianza condicionada: Var Y|X ( Y | a) =


y RY

y E Y|X ( Y | a)

P Y|X ( y | a)
2

=E Y|X Y 2 a E Y|X ( Y | a)

Ejercicio 24. Sea la siguiente funci on PXY (x, y ) x=0 x=1 Calcule E X|Y ( X | 2). Soluci on : PXY (x, y ) x=0 x=1 Marginal de Y y=0 0.33 0 0.33 y=1 0.12 0.25 0.37

de cuant a conjunta para X e Y y=0 y=1 y=2 0.33 0.12 0.05 0 0.25 0.25

y=2 0.05 ; 0.25 0.3

por tanto

P X|Y ( X | 2) x=0 x=1

0.05/0.3 0.25/0.3 Ejercicio 24

y entonces E X|Y( X | 2) = 0 0.05/0.3+1 0.25/0.3 = 0.25/0.3 = 0.83

Momentos condicionados: caso continuo

54

Esperanza condicionada: E Y|X ( Y | a) =


y RY

y f Y|X ( y | a) dy

Varianza condicionada: Var Y|X ( Y | a) =


y RY

y E Y|X ( Y | a)

f Y|X ( y | a) dy
2

=E Y|X Y 2 a E Y|X ( Y | a)

Ejercicio 25. Sea la siguiente funci on de densidad condicionada ( kx si 0 < y < x, para x (0, 1) f Y|X ( Y | x) = . 0 en el resto de casos C omo calcular a el valor esperado de Y condicionado a que X tomase un valor arbitrario en el intervalo (0,1)? (NO lo calcule, pero indique correctamente los ordenes de integraci on). Soluci on : E Y|X ( Y | x) =
0

Z y f y|X ( y | x) dy =
0

kxy dy Ejercicio 25

9.1. Propiedades de la esperanza condicional

Propiedades de la esperanza condicional

55

Para X , Y y Z v.a. y a y b constantes; E | ( | ) satisface las siguientes propiedades:

EC1. [Linealidad] E XY|Z ( aX + bY | z) = a E X|Z ( X | z) + b E Y|Z ( Y | z)

EC2. [Sustituyendo lo conocido] E Y|X ( h(X, Y ) | x) = E Y|X ( h(x, Y ) | x)

EC3. Si X e Y independientes:

E Y|X ( g (Y ) | x) = E (g (Y ))

Ejercicio 26. La variable aleatoria bidimensional (X ,Y ) tiene funci on bidimensional conjunta ( k si 0 < y < 1 x < 1 fXY (x, y ) = 0 en el resto de los casos

(a) Calcule k para que fXY (x, y ) sea una verdadera funci on de densidad Soluci on: El soporte en un tri angulo rect angulo cuyos catetos son los ejes x e y que se unen en el origen, y cuya hipotenusa es el segmento que va de (x, y ) = (0, 1) hasta (x, y ) = (1, 0) (y que corresponde a la funci on y = 1 x).
1


y = 1 x

Para que sea funci on de densidad debe integrar uno: 1 Z 1 Z 1 Z 1 Z 1y y2 y k dxdy = k [x]1 dy = k 1 y dy = k y = k/2 = 1 0 2 0 0 0 0 0 por lo tanto, k = 2. Ejercicio 26

(b) P (X < 0.1, Y > 0.8). Soluci on: Debemos calcular la probabilidad asociada al siguiente recinto

.8

.1

es decir Z P (X < 0.1, Y > 0.8) =


0 0.8 0.1 0.1

1x

Z 2dy dx =
0

0.1

1x 2y 0.8 dx

Z =
0

2 (1 x) 2 (0.8) dx
0.1

Z =
0

0.4 2x dx

0.1 = 0.4x x2 0 dx

=0.4 (0.1) (0.1)2 = 0.03

Ejercicio 26

(c) Calcule las funciones de densidad marginales, as como sus esperanzas. Soluci on: Para X Z 1x Z fX (x) = fXY (x, y ) dy =
0 0 1x

1x 2dy = 2y 0 = 2 2x

si 0 < x < 1

y fX (x) = 0 en el resto de los casos. Para Y Z 1y Z 1y 1y fY (y ) = fXY (x, y ) dx = 2dx = 2x 0 = 2 2y


0 0

si 0 < y < 1

y fY (y ) = 0 en el resto de los casos. Por tanto 1 Z 1 Z 1 2 x3 xfX (x) dx = 2x 2x2 dx = x2 =1 E (X ) = 3 0 0 0 Z 1 Z 1 1 2y 3 E (Y ) = yfY (y ) dy = 2y 2y 2 dy = y 2 =1 3 0 0 0

2 1 = 3 3 2 1 = 3 3

Ejercicio 26

(d) Calcule E X|Y ( X | y). Es igual a la esperanza incondicional de X ? Soluci on: Primero debemos calcular la funci on de densidad condicionada fXY (x, y ) 2 f X|Y ( X | y) = = fY (y ) 2 2y por tanto, 2 1y Z 1y Z 1y 1 1 1 x 1y E X|Y ( X | y) = xdx = x dx = = 1y 1y 0 1y 2 0 2 0
1


y = 1 x

Que es distinta de la esperanza incondicional, que es igual a 1/3. Ejercicio 26 (e) Para que valor de Y la esperanza condicional, E X|Y ( X | y), es igual a E (X ). Soluci on : 1y 1 = 2 3 y= 1 3 Ejercicio 26

(f) Son X e Y independientes? Soluci on: Puesto que la esperanza condicional es distinta de la incondicional, estas variables no son independientes. Ejercicio 26 (g) Calcule la covarianza entre X e Y . Comente el resultado. Soluci on: Para ello hemos de calcular E (XY ). Z 1 Z 1x Z E (XY ) = xyfXY (x, y ) dydx =
0 0 1 0 0

Z
0

1x

xy 2dydx

Z = Z =
0

2xy 2 2

1x dydx =
0

Z
0

x(1 x)2 dx 1 =
0

(x 2x2 + x3 )dx =

x2 2 x3 x4 + 2 3 4

1 12

Por lo tanto Cov (X, Y ) = E (XY ) E (X ) E (Y ) = 1 11 1 = 12 33 36

Es decir, hay una relaci on lineal decreciente entre X e Y como cab a esperar dado dado el soporte de la funci on de densidad conjunta, que es uniforme (n otese la relaci on lineal decreciente de la esperanza condicional, i.e., la funci on de regresi on). Ejercicio 26

(h) Calcule Var X|Y ( X | y) Soluci on : ` E X|Y X 2 y = Z


0 1y 1y

x2

1 1 dx = 1y 1y

Z
0

xdx = ` X2

1 1y

x3 3

1y =
0

(1 y )2 ; 3

y [E X|Y ( X | y)]2 as pues, puesto que Var X|Y ( X | y) = E X|Y (1 y )2 (1 y )2 1y 2 = Var X|Y ( X | y) = 3 2 12 para 0 < y < 1; por tanto, la varianza var a entre cero para y = 1 y
1 12

para y = 0. Ejercicio 26

Ejercicio 27. Sean X e Y dos variables aleatorias discretas. Si E Y|X ( Y | 1) = 1 y adem as Var Y|X ( Y | 1) = 0. Cual es el valor de P Y|X ( 1 | 1)? y el valor de P Y|X ( 2 | 1)? Justique su respuesta. Soluci on: Que Var Y|X ( Y | 1) = 0 implica que cuando X = 1, la variable aleatoria Y es degenerada, es decir, toma un valor constante. Puesto que E Y|X ( Y | 1) = 1, dicho valor es uno. Por lo tanto P Y|X ( 1 | 1) necesariamente es igual a 1. De esto se deduce que la probabilidad de que Y tome un valor distinto de 1 condicionado a X =1 es cero; en particular P Y|X ( 2 | 1) = 0. Ejercicio 27

Ejercicio 28. [Ejemplo 7.1 (Novales, 1997, pp. 247)] Sea X e Y con una funci on de cuant a conjunta denida sobre el soporte RXY = {(2, 4), (1, 1), (0, 0), (1, 1), (2, 4)} que asigna a cada punto una probabilidad de 1/5.

(a) Comprobar si X y Y son independientes. Soluci on: La funci on de cuant a marginal de X es: 8 > 1/5 para x = 2 > > > > > >1/5 para x = 1 < P X (x) = 1/5 para x = 0 > > > > 1/5 para x = 1 > > > :1/5 para x = 2

mientras que la funci on de cuant a marginal de 8 > > <1/5 para P Y (y ) = 2/5 para > > :2/5 para Puesto que

Y es : y=0 y=1 y=4 .

PXY (1, 1) = 1/5 y P X (1) P Y (1) = no son independientes. 1 2 2 = , 5 5 25 Ejercicio 28

(b) Calcular Cov (X, Y ) Soluci on: Por una parte E (XY ) = 8 por otra E (X ) = 2 y E (Y ) = 0 As pues, Cov (X, Y ) = E (XY ) E (X ) E (Y ) = 0 0 2 = 0 Ejercicio 28 1 2 2 + 1 + 4 = 2. 5 5 5 1 1 1 1 1 1 +0 +1 +2 =0 5 5 5 5 5 1 1 1 1 1 1 + 0 + 1 + 8 = 0; 5 5 5 5 5

(c) (a nadido) Calcular 1. la funci on de cuant a de X condicionada a Y = 0 2. la funci on de cuant a de X condicionada a Y = 1 3. la funci on de cuant a de X condicionada a Y = 4 Soluci on : n 1. P X|Y ( x | 0) = 1 para x = 0 ( 1/2 para x = 1 2. P X|Y ( x | 1) = 1/2 para x = 1 ( 1/2 para x = 2 3. P X|Y ( x | 4) = 1/2 para x = 2 Ejercicio 28 (d) (a nadido) Calcular la funci on de cuant a de Y condicionada a X = 0. n Soluci on: P Y|X ( y | 0) = 1 para y = 0

Ejercicio 28

(e) (a nadido) Calcular 1. E X|Y ( X | 0) 2. E X|Y ( X | 1) 3. E X|Y ( X | 4) Soluci on : 1. E X|Y ( X | 0) = 0 P X|Y ( 0 | 0) = 0 1 = 0 2. E X|Y ( X | 1) = 1 P X|Y ( 1 | 1) + 1 P X|Y ( 1 | 1) = = 1 1/ 2 + 1 1/ 2 = 0 3. E X|Y ( X | 4) = 2 P X|Y ( 2 | 4) + 2 P X|Y ( 2 | 4) = = 2 1/ 2 + 2 1/ 2 = 0 Ejercicio 28 (f) (a nadido) Calcular E Y|X ( Y | 0). Soluci on : E Y|X ( Y | 0) = 0 P X|Y ( 0 | 0) = 0 1 = 0 Ejercicio 28

(g) (a nadido) Calcular 1. Var X|Y ( X | 0) 2. Var X|Y ( X | 1) 3. Var X|Y ( X | 4) Soluci on : 1. Var X|Y ( X | 0) = (0 0)2 P X|Y ( 0 | 0) = (0 0)2 1 = 0 2. Var X|Y ( X | 1) = (1 0)2 P X|Y ( 1 | 1) + (1 0)2 P X|Y ( 1 | 1) = 1 1 /2 + 1 1 /2 = 1 3. Var X|Y ( X | 4) = (2 0)2 P X|Y ( 2 | 4) + (2 0)2 P X|Y ( 2 | 4) = 4 1 /2 + 4 1 /2 = 4 agina163. Ejercicio 28 Continuaci on en Ejercicio 29 en la p

10. Funci on de regresi on y funci on ced astica. Esperanzas y varianzas condicionales estoc asticas

Esperanza condicional como funci on de x: E Y|X ( Y | x)

56

Funci on de regresi ona es E Y|X ( Y | xj ) interpretada como funci on de xj para xj RX :

h(x) = E Y|X ( Y | x) ,

x RX

a de

Y sobre X

Esperanza condicional como v.a.: E (Y | X )

57

Esperanza condicional estoc astica: v.a. que toma los valores h(x) = E Y|X ( Y | xj ) , xj RX

con probabilidades determinadas por la ley de probabilidad de X : E (Y | X ) = E Y|X ( Y | xj ) con probabilidades provenientes de FX (xj ) para todo xj RX

Es decir, si h(x) = E Y|X ( Y | x) es la funci on de regresi on, entonces h(X ) E (Y | X ) es la esperanza condicional estoc astica.

Ejercicio 29. (a) Represente la funci on de regresi on E X|Y ( X | y) del Ejercicio 28. (b) Cual es la funci on de cuant a de E (X | Y )? Soluci on : P (E (X | Y )) = n 1 para E (X | Y ) = 0 Ejercicio 29

Varianza condicional como funci on de x: Var Y|X ( Y | x)

58

Funci on ced asticaa es Var Y|X ( Y | x) interpretada como funci on de x: g (x) = Var Y|X ( Y | x) , x RX

Var Y|X ( Y | x) = cte Var. cond. homoced astica. Var Y|X ( Y | x) = g (x) = cte Var. cond. heteroced astica.

a de

Y sobre X

Varianza condicional como v.a.: Var (Y | X )

59

Varianza condicional estoc astica: v.a. que toma los valores Var Y|X ( Y | xj ) , xj RX

con probabilidades determinadas por la ley de probabilidad de X : Var (Y | X ) = Var Y|X ( Y | xj ) con probabilidades provenientes de FX (xj ) para todo xj RX

Es decir, si g (x) = Var Y|X ( Y | x) es la funci on ced astica, entonces g (X ) = Var (Y | X ) = E Y 2 X [E (Y | X )] es la varianza condicional estoc astica.
2

Ejercicio 30. (a) Represente la funci on ced astica E X|Y ( X | y) del Ejercicio 28. (b) Cual es la funci on de cuant a de Var (X | Y )? Soluci on : 8 > > < 1/5 P (Var (X | Y )) = 2/5 > > : 2/5 para Var (X | Y ) = 0 para Var (X | Y ) = 1 para Var (X | Y ) = 4 Ejercicio 30

10.1. Esperanzas iteradas

Teorema de las esperanzas iteradas

60

E X (E (Y | X )) = = = = =

h(x)P X (x) [E Y|X ( Y | x)] P X (x) [ yP Y|X ( y | x)] P X (x) y P Y|X ( y | x) P X (x) y PXY (x, y ) = E (Y )

10.2. Identidad de la varianza condicional

Identidad de la varianza condicional

61

Var (Y ) = E (Var (Y | X )) + Var (E (Y | X )) que implica Var (Y ) Var (E (Y | X ))

Ejercicio 31. Sea (X, Y ) una variable aleatoria bidimensional continua con la siguiente funci on de densidad: ( 2/3 si x [0, 1]; y [0, 2]; y>x fXY (x, y ) = 0 en los restantes casos

(a) Dibuje el recinto en el que fXY (x, y ) toma valores distintos de cero. Especique claramente los rangos de denici on (incondicionales y condicionales) de las variables aleatorias X e Y . (valores tomados por y restringen en cualquier caso los posibles valores que puede tomar x?) Soluci on :
y 2

1
y= x

N otese que x puede tomar valores entre 0 y 1, siempre y cuando adem as x < y . Cuando y < 1, la variable x estar a en el intervalo x (0, y ); por lo que y restringe los posibles valores de x. Sin embargo, cuando y 1, la variable x puede tomar cualquier valor entre cero y uno (y NO restringe, en este segundo caso, los posibles valores de x.) Ejercicio 31

(b) Verique que fXY (x, y ) = 2/3 es efectivamente una funci on de densidad denida en el recinto especicado. R1R2 as fXY (x, y ) es estrictamente positiva en todo Soluci on : 0 x 2/3 dydx = 1 y adem su dominio de denici on. Ejercicio 31

(c) Calcule las funciones de densidad marginal de X e Y . Indique expl citamente sus dominios de denici on. Calcule el valor esperado de X e Y . Soluci on : (R 2 fX (x) =
x

2/3 dy =

4 3

2 x, 3

en el intervalo x (0, 1) en el resto de casos.

0 8R y > > 0 2/3 dx = < R1 fY (y ) = 0 2/3 dx = > > :0

2 y, 3 2 , 3

en el intervalo y (0, 1) en el intervalo y [1, 2) en el resto de casos.

Los valores esperados son: Z E (Y ) =


0

2 y y dy + 3 x

Z
1

2 11 dy = . 3 9 4 . 9 Ejercicio 31

Z E (X ) =
0

2 4 x 3 3

dx =

(d) Verique que la probabilidad de que Y sea mayor que 1 es dos tercios, y la probabilidad de que sea menor o igual que 1 un tercio. Soluci on : Z 2 2 2 dy = ; 3 1 3 Z 1 1 2 ydy = ; 3 0 3

como cabe esperar por ser el suceso complementario al anterior. Ejercicio 31

(e) Calcule la funci on de regresi on de Y sobre X (esperanza de Y condicionada a X ) y la funci on de regresi on de X sobre Y (la esperanza de X condicionada a Y ). Deje bien claro los dominios de denici on de las dos esperanzas condicionadas. Dibuje ambas esperanzas condicionadas. Soluci on : 82 R > > > < y
x 2/3
42x 3 3

dy =

x2 2x4

2 x2

1 2

(x + 2) ,

denida para x (0, 1),

E Y|X( Y | x) =

> > > :

0
y 2

en otro caso.

1
y= x

8R y 2/3 1 > > 0 x 2/3y dx = 2 y > > > > > < R 1 2/3 E X|Y ( X | y) = x dx = 1 2 > 0 2/3 > > > > > > : 0
y 2

denida para y (0, 1), denida para y [1, 2), en otro caso.

1
y= x

Ejercicio 31

(f) Son lineales las funciones de regresi on (las esperanzas condicionadas) calculadas en el apartado anterior? Soluci on: E Y|X ( Y | x) es lineal; sin embargo E X|Y ( X | y) no lo es. Ejercicio 31

Esperanza como predictor optimo

10.3. Propiedades de la esperanza condicional estoc astica

Propiedades de la esperanza condicional estoc astica

62

Para X , Y y Z v.a. y a y b constantes; E ( | ) satisface las siguientes propiedades: ECS1. [Linealidad] E(aX + bY |Z ) = aE (X | Z) + bE (Y | Z) ECS2. [Ley de esperanzas iteradas] E (E(g (X, Y )|X )) = E (g (X, Y )) ECS3. [Sacando fuera lo conocido] E( h(X ) g (Y ) |X ) = h(X ) E(g (Y )|X ) ECS4. [Predictor optimo] E [Y E (Y | X )] cualquier g (X )
2

E [Y g (X )]

para

Esperanza condicional como predictor optimo

63

Buscamos una funci on de X , digamos h(X ), tal que E XY sea m nimo.

Y h(X )

E XY

Y h(X )

= E X E Y|X

Y h(x)

Buscamos h(x) que hace E Y|X

Y h(x)

nimo para todo x RX . x m

Esperanza condicional como predictor optimo

64

+ Var Y|X ( Y | x)
cte.

que es m nimo cuando h(x) = E Y|X ( Y | x) .

10.4. Esperanza condicional cuando es una funci on lineal

Sea E (Y | X ) una funci on lineal de X , es decir, E (Y | X ) = a + bX C omo son en este caso a y b en relaci on a los momentos te oricos de fXY (x, y )? De ECS1 y ECS2 sabemos que E (Y ) = E X (E Y|X ( Y | x)) = E (a + bX ) = a + bE (X ) ; por tanto, a = E (Y ) b E (X ) (10.1)

De ECS2 y ECS3 E (XY ) = E X (E (XY | X )) = E (X E (Y | X )) por tanto E (XY ) =E (X [a + bX ]) h i =E X E (Y ) b E (X ) + bX ` =E (X E (Y )) + b E X 2 E (X ) E (X ) =E (Y ) E (X ) + b Var (X ) ; despejando b Var (X ) : b Var (X ) = E (XY ) E (X ) E (Y ) Cov (X, Y ) . Var (X )

de 10.1

b=

(10.2)

10.5. Aproximaci on lineal de la esperanza condicional. Recta de regresi o

Aproximaci on lineal a la esperanza condicional: Recta de regresi on

65

E (Y ) es un predictor de Y h(x) = E Y|X ( Y | x) es predictor preferido (incorpora informaci on relevante cuando Y y X dependientes) Si conocemos fXY (x, y ) podemos deducir h(x) = E Y|X ( Y | x) En general desconocemos tanto fXY (x, y ) como h(x) = E Y|X ( Y | x) (aunque con frecuencia conocemos realizaciones de X ) Una soluci on es buscar una funci on g (x) alternativa a la funci on h(x) desconocida, pero que se parezca a h(x) en alg un sentido.

Aproximaci on lineal a la esperanza condicional: Recta de regresi on

66

Buscamos la aproximaci on m nimo cuadr atica (que minimiza la suma ponderada de la distancia al cuadrado):

m n
g RX

[g (x) h(x)] P X (x) ,

donde fX (x) y h(x) = E Y|X ( Y | x) son funciones desconocidas. Si suponemos g (x) = a + bx :

m n
g RX

[(a + bx) h(x)] P X (x)

Aproximaci on lineal a la esperanza condicional: Recta de regresi on

67

Condiciones de primer orden (par ametro a):

2
RX

[a + b x h(x)] P X (x)

a
RX

P X (x) + b
RX

xP X (x)
RX

E Y|X ( Y | x) P X (x)

a + b E (X ) E (Y )

a = E (Y ) b E (X )

Aproximaci on lineal a la esperanza condicional: Recta de regresi on

68

Condiciones de primer orden (par ametro b): 2


RX

[a + b x h(x)] xP X (x)

= =

0 0

aE (X ) + bE X 2 E (X E (Y | X )) donde E (X E (Y | X )) =
RX

x
RY | X

PXY (x, y ) P X (x) = E (XY ) P X (x)

y sustituyendo el valor de a que obtuvimos anteriormente, b = Cov (X, Y ) , Var (X ) E (Y ) E (X ) Cov (Y, X ) . Var (X )

a =

Aproximaci on lineal a la esperanza condicional: Recta de regresi on

69

g (x) = E (Y ) E (X )

Cov (Y, X ) Cov (X, Y ) + x; Var (X ) Var (X )

x RX , (10.3)

y puesto que XY = expresi on alternativa:

XY X Y

entonces

XY 2 X

= XY

Y X

y tenemos una

g (x) = E (Y ) + XY ona . de la recta de regresi


a pero

Y (x E (X )); X

x RX ,

(10.4)

que depende de momentos que en general son desconocidos!

Ejemplo 3. Sean las siguientes v.a. referentes a una empresa Y tasa de variaci on de los benecios X tasa de variaci on de los gastos en personal Z tasa de variaci on de los gastos en publicidad Conocemos los siguientes momentos de dichas variables E (Y ) = 0.04 E (X ) = 0.03 E (Z ) = 0.035 Y = 0.01 X = 0.015 Z = 0.025 XY = 0.5 ZY = 0.6

Podemos incrementar en un 5 % una de las dos vbles de control (bien X , o bien Z ) Que decisi on es preferible?

Empleando la aproximaci on lineal a la esperanza condicional (regresi on lineal) vemos que, por una parte: E Y|X ( Y | x) ax + bx x,
0.01 Y = 0.5 0 = 0.333 (que es la pendiente) y ax = E(Y ) E(X ) bx = donde bx = XY .015 X 0.04 0.03 0.333 = 0.03. Por lo tanto E Y|X ( Y | x) 0.03 + 0.333 x,

por lo que el incremento esperado en los benecios condicionado a un incremento del 5 % en el gasto en personal es: E Y|X ( Y | 0.05) 0.03 + 0.333 0.05 = 0.0466, Por otra parte E Y|Z ( Y | z) az + bz z,
0.01 Y donde bz = ZY = 0.6 0 = 0.24 (que es la pendiente) y az = E (Y ) E (Z ) bx = .025 Z 0.04 0.035 0.24 = 0.0316. Por lo tanto E Y|Z ( Y | z) 0.0316 + 0.24 z,

por lo que el incremento esperado en los benecios condicionado a un incremento del 5 % en el gasto en publicidad es: E Y|Z ( Y | 0.05) 0.0316 + 0.24 0.05 = 0.0436, Pese a que ZY > XY , la pendiente de la funci on de regresi on de Y sobre X es mayor que la pendiente de la funci on de regresi on de Y sobre Z ; por lo que un incremento del gasto del 5 % en personal arroja un mayor benecio esperado que en el caso de que dicho incremento se produzca en publicidad.

Ejercicio 32. (Consta de 9 apartados) Sea (X ,Y ) una variable aleatoria bidimensional continua con funci on de densidad conjunta fXY (x, y ) denida sobre el soporte 1 y x; 0x1

Deje indicado c omo calcular a las siguientes cuestiones: (En este ejercico no se le pide realizar calculos, tan s olo plantear y/o expresar adecuadamente lo requerido en cada apartado; por ello preste especial atenci on en indicar cl aramente los l mites de integraci on y dominio de las funciones de densidad en cada caso; no hacerlo signicar a no haber respondido a la pregunta! ) (a) Dibuje el soporte de fXY (x, y ). Soluci on :
y

1 0
y = x

Ejercicio 32

(b) C omo vericar a que fXY (x, y ) es funci on de densidad? 1. Primero integrando respecto de y y despu es respecto de x 2. Primero integrando respecto de x y despu es respecto de y Soluci on: Comprobando si se verican las siguientes igualdades 1. Z 1Z x fXY (x, y ) dy dx = 1
0 1

2. Z
0

1Z

Z fXY (x, y ) dx dy +

0 1

Z
0

fXY (x, y ) dx dy = 1

Ejercicio 32 (c) Las funciones de densidad marginales de X y Y , especicando explicitamente sus soportes (dominio de cada funci on de densidad). Soluci on : Z fX (x) =
1 x

fXY (x, y ) dy ; 8R 1 > > < y fXY (x, y ) dx; > > :R 1 f
0
XY

x [0, 1] y (0, 1] y [1, 0] Ejercicio 32

fY (y ) =

(x, y ) dx;

(d) C omo vericar a si X e Y son independientes o no? Soluci on: Deber a comprobarse si se verica fXY (x, y ) = fX (x) fY (y ) para 1 y x; en caso armativo, X e Y son independientes. (e) La Varianza de X 2 ` Soluci on: Puesto que Var (X ) = E X 2 E (X ) , entonces Z Var (X ) =
0 1 1

0x1 Ejercicio 32

x2 fXY (x, y ) dy dx

Z
0

x 1

2 xfXY (x, y ) dy dx

o lo que es lo mismo Z Var (X ) =


0 1

x2 fX (x) dx

Z
0

2 xfX (x) dx Ejercicio 32

(f) La esperanza de Y condicionanda a X = x. Soluci on: Para ello es necesario calcular la funci on de densidad condicionada, que en este caso es fXY (a, y ) f Y|X ( y | a) = ; 1 y a fX (a) por lo tanto Z x fXY (x, y ) dy ; x [1, 0]; 1 < y < x. E Y|X ( Y | x) = y fX (x) 1 Ejercicio 32 (g) Cual ser a la expresi on exacta de la anterior esperanza condicional en el caso de ser una funci on lineal de x. Soluci on : E Y|X ( Y | x) = E (Y ) E (X ) Cov (Y, X ) Cov (X, Y ) + x; Var (X ) Var (X ) x [0, 1]. Ejercicio 32

(h) P ( Y > d | X = 0.5), donde 1 d 1. Soluci on : Z P ( Y > d | X = 0.5) =


d 0.5

Z f Y|X ( y | 0.5) dy =

0.5 d

fXY (0.5, y ) dy fX (0.5)

si d < 0.5

P ( Y > d | X = 0.5) =0 (i) P ( Y > d | X > 0.5), donde 1 d 1.

si d 0.5 Ejercicio 32

Soluci on: En este caso no podemos emplear la funci on de densidad condicionada f Y|X( Y | x); aqu debemos emplear directamente la denici on de probabilidad condicionad R1 Rx d fXY (x, y ) dy dx P ( Y > d | X > 0.5) = 0.5 R si d < 0.5 1 0.5 fX (x) dx R1Rx P ( Y > d | X > 0.5) =
d d

fXY (x, y ) dy dx R1 0.5 fX (x) dx

si d 0.5 Ejercicio 32

Ejercicio 33. Un inversor desea saber si le intreresa invertir en bolsa. Este inversor sabe que la ley de probabilidades conjunta de las rentabilidades del IBEX35 (X ) y del tipo de inter es de los bonos a un a no (Y ) es PXY (x, y ) = k(1 XY ); donde x 1, 0, 1 es una variable que toma el valor 1 si la rentabilidad es negativa (p erdidas), 0 si la rentabilidad es nula y 1 hay rentabilidades positivas; y por otra parte y 1, 0, 1 es una variable que vale 1 si hay una bajada de tipos de inter es, 0 si los tipos se mantienen y 1 si los tipos suben (a) Encuentre el valor de k para que PXY (x, y ) sea una funci on de cuant a. (b) Calcular la rentabilidad esperada independientemente de lo que ocurra con los tipos de inter es. (c) Calcule la rentabilidad esperada si el tipo de inter es cae. (d) Calcule el coeciente de correlaci on (e) Si denimos la prima al riesgo del mercado como el diferencial de rentabilidad respecto a los tipos: Z = X Y, calcule la prima esperada. (f) Si denimos la prima al riesgo del mercado como el diferencial de rentabilidad respecto a los tipos: Z = X Y, calcule la prima esperada en un escenario de bajada de tipos de inter es. (g) Calcule el valor de a y b tales que E Y|X ( Y | x) a + bx

11. Transformaci on de variables

Transformaci on de una v.a. unidimensional

70

Sea X con fX (x) sobre RX .

Sea la funci on h() continua, diferenciable (y mon otonaa )

Deseamos conocer la distribuci on de la v.a. denida como Y = h(X )

a si no es mon otona, el problema es muy similar pero ligeramente m as conplejo (Papoulis, 1991, cap. 5)

Transformaci on de una v.a. unidimensional

71

Sea Y = h(X ); entonces FY (b) = P (Y b) = P (h(X ) b) si h() es mon otona creciente, y aplicando h()1 ; FY (b) = P X h1 (b) = FX h1 (b)

y b

h(x)

h1 (b)

Transformaci on de una v.a. unidimensional

72

Sea Y = h(X ); entonces FY (b) = P (Y b) = P (h(X ) b) si h() es mon otona decreciente, y aplicando h()1 ; FY (b) = P X h1 (b) = 1 P X h1 (b) = 1 FX h1 (b)
y

b
h(x)

h1(b)

Transformaci on de una v.a. unidimensional

73

Si h() es creciente la funci on de densidad de la v.a. transformada es dFX h1 (y ) dFY (y ) dh1 (y ) = fY (y ) = = fX h1 (y ) dy dy dy y si h() es decreciente d(1 FX h1 (y ) ) dFY (y ) dh1 (y ) = fY (y ) = = fX h1 (y ) dy dy dy En general dFY (y ) dh1 (y ) = fY (y ) = fX h1 (y ) dy dy nota: en Novales (1997) se sustituye h1 (y ) por x.

Transformaci on de una v.a.: ejemplo

74

Sea fX (x) = 1 en el intervalo x (0, 1) y cero en el resto.

Sea Y = 3X 1;

entonces
y +1 3

FY (y ) = P (Y y ) = P (3X 1 y ) = P X = FX y por tanto fY (y ) =


y +1 3

dFX h1 (y ) dh1 (y ) 1 = fX h1 (y ) =1 dy dy 3

en el intervalo y h(0), h(1) = (1, 2), y cero en el resto.

Transformaci on de una v.a.: ejemplo

75

fX (x) 1

fY (y ) 1/3 0 1 -1 0 2

FX (x) 1

FY (y ) 1

-1

Esp. condicional como funci on de regresi on o como v.a.: utilidad

76

El BCE desea conocer la probabilidad de que la inaci on, supere el 2 %.

El BCE sabe que la inaci on depende del nivel de los tipos de inter es o del volumen de ALPs.

Esp. condicional como funci on de regresi on o como v.a.: utilidad

77

El BCE puede jarse en lo siguiente 1. Calcular P ( > 0.02) (c alculo pobre, poca informaci on en la distribuci on marginal) 2. Puede intentar emplear la funci on de distribuci on condicional (pero necesita la funci on de densidad conjunta que en general es desconocida) 3. Puede emplear la aproximaci on lineal a la esperanza condicional (solo necesita estimar algunos par ametros) Pero le d a valores para un nivel de ALP dado (que en general es desconocido) 4. Puede (alternativamente) calcular P (E ( | ALP ) > 0.02) Necesito suponer alguna dist. sobre ALP y el tipo funcional de E |ALP ( | alp).(generalmente supondremos linealidad)

Fun. de densidad de la esperanza condicional: suponiendo linealidad

78

Supongamos que E Y|X ( Y | x) es funci on lineal de x E Y|X ( Y | x) = a + bx = h(x)a ; entonces la funci on de densidad de E (Y | X ) = h(X ) es

fE(Y |X ) (y )

dh1 (y ) dy

fX h1 (y )

1 b

fX

y a b

ya que de y = a + bx se deduce que x =

y a b .

a por

lo que h1 (y ) =

y a b

Ejercicio 34. Sea X Uniforme (0, 1) . Sea Z = E (Y | X ) = 2+ 3X . Halle la funci on fZ (z ) (explicitando su soporte RZ ). Halle E (Z ). Qu e probabilidad hay de que el valor esperado de Y condicionado a X sea mayor que 3? Pista. la funci on de densidad de X Uniforme (0, 1) es fX (x) = 1, 0 x 1. Soluci on: Puesto que Z = 3X + 2; entonces FZ (z ) = P (Z z ) = P (3X + 2 z ) = P y 1 dh (z ) = 1 1; fZ (z ) = fX (h1 (z )) dz 3 ` z RZ = h(0), h(1) = (2, 5) R3
2 1 z3 dz =

z2 3

= FX

z2 3

Es decir, Z Uniforme (2, 5) . Por tanto, E (Z ) =

Puesto que Z = E (Y | X ), entonces P (E (Y | X ) > 3) = P (Z > 3) : Z 5 1 z 2 53 5 P (Z > 3) = dz = = = 3 3 3 3 3 3 Ejercicio 34

3 1 z2 3 2 2

7 . 2

Ejercicio 35. Sea la funci on de densidad bivariante ( xy si 0 < y < x < 4 fXY (x, y ) = 32 . 0 en el resto de casos (a) Dibuje el soporte de la distribuci on. Soluci on :
y 4
y = x

y0

x0

Ejercicio 35

(b) Calcule las funciones de densidad condicionadas de X e Y . Soluci on: Puesto que las funciones de densidad marginales son R 4 xy 3 fY (y ) = y dx = y y para y (0, 4). 32 4 64 fX (x) = Rx
xy 0 32

dy =

x3 , 64

para x (0, 4).

entonces 1. f x|Y ( x | y) =

xy 32 3 y y 4 64 xy 32 x3 64

2x , (16y 2 )

y < x < 4;

es decir R Y | X = (y, 4)

2. f y|X ( y | x) =

2y , x2

0 < y < x;

es decir R Y | X = (0, x) Ejercicio 35

(c) Calcule el valor esperado de Y . R4 1 1 3 y ( 4 y 64 y )dy = Soluci o n : E (Y ) = 0

32 15

= 2.13

Ejercicio 35

(d) Calcule y represente gr acamente E Y|X ( Y | x). Cuando X = 2, cu al es el valor esperado de Y ? Coincide con la esperanza incondicional? Soluci on: Para cualquier x (0, 4), f Y|X ( Y | x) = Entonces E Y|X ( Y | x) = Rx
0 1 xy 32 1 3 x 64

2y ; x2

y (x, 4).

2y yx 2 dy =

2 x, 3

para x (0, 4).

La esperanza condicionada est a representada por la linea azul.


y 4
y = x

y0

x0

Si X =2, E (Y |X = 2) = 4/3 < E (Y ) . Por tanto no coinciden.

Ejercicio 35

(e) Son X e Y independientes? Soluci on: En este caso fXY (x, y ) = fX (x) fY (y ) ; pero ya no es necesario vericarlo puesto que E Y|X ( Y | x) diere de E (Y ), ambas variables dependen. Ejercicio 35 (f) A partir de qu e valor de X , el valor esperado de Y condicionado a X es mayor que el valor esperado de Y incondicional. x > 32 , es decir, cuando x > 16 . Por Soluci on: E Y|X ( Y | x) > E (Y ) s olo cuando 2 3 15 5 lo tanto, si x (16/5, 4) entonces E Y|X ( Y | x) > E (Y ) . Ejercicio 35 (g) Calcule, por tanto, la probabilidad P (E (Y | X ) > E (Y )) . ` Soluci on: Seg un el apartado anterior, es la probabilidad de que P X > 16 . Por 5 tanto Z 4 Z 4 1 3 369 16 = fX (x) dx = x dx = = 0.59 P X> 5 625 16/5 16/5 64 Ejercicio 35

(h) Calcule la funci on de densidad de E (Y | X ) Soluci on: Puesto que E Y|X ( Y | x) = y = h ( x) = adem as fX (x) =
1 3 x , 64 2 x, 3

entonces RE (Y

|X )

= (0, 8/3) y 3 y; 2

2 x 3

y por tanto

x = h1 (y ) =

as pues 3 3 81 3 3 fX 3 y = 3 1 = fE (Y |X ) (y ) = y y 2 2 2 64 2 1024
8 ) 3

para y (0, Soluci on :

Ejercicio 35

(i) Con la nueva informaci on verique que P (E (Y | X ) > E (Y )) = 0.59


8/3 E(Y )

Z P (E (Y | X ) > E (Y )) = P (E (Y | X ) > 32/15) =

81 3 y = 0.59 1024 Ejercicio 35

Transformaci on de una v.a. bidimensional

79

Transformaci on de una v.a. bidimensional Sean X e Y con fXY (x, y ) . Y sean las variables aleatorias U = g1 (X, Y ) y V = g2 (X, Y ), con inversas X = h1 (U, V ) e Y = h2 (U, V ). Entonces fUV (u, v ) = |J | fXY (h1 (U, V ), h2 (U, V )) , donde J det
h1 (u,v ) u h2 (u,v ) u h1 (u,v ) v h2 (u,v ) v

x u y u

x v y v

y donde

x u

h1 (u,v ) , u

x v

h1 (u,v ) , v

Ejemplo 4. Sea fXY (x, y ) = 4xy ; Sean U =X +Y ; V =Y ; entonces 0 v 1,


v

0 x 1,

0 y 1.

0 v u 1 + v 2.

u = = v u

+ 1 = u

Las funciones inversas son Y = V ; 1 1 y J = 0 1

X = U Y = U V.

Por tanto

fUV (u, v ) = |J | fXY (x, y ) = 4(u v )v ;

donde 0 v 1, y v u 1 + v.

La funci on de densidad de X +Y es la marginal de U ; por tanto (v ease la gura): h 2 i 8 Ru 3 u 2 3 4 v (u v )dv = 4 u v v = 3 u ; si 0 u 1 > > 2 3 0 > 0 > > < fU(u) = . > > > h 2 i1 > R 3 > :4 1 v (u v )dv = 4 u v v = 1 (12u 2u3 8); si 1 u 2 u1 2 3 3
u1

Ejemplo 5. Sean X1 y X2 indep Exponencial (), es decir, fX (x) = ex ; Demostrar que Y1 = X1 + X2 y Y2 = X1 /X2 son variables independientes: T : (x1 , x2 ) (y1 , y2 ); Por tanto T 1 ser a: x2 = y 1 x1 ; x2 = y 1 x2 y 2 x2 (1 + y2 ) =y1 x2 = y1 ; 1 + y2 x1 = y1 y2 , 1 + y2 y x1 = y 2 x2 donde y 1 = x1 + x2 , y2 = x1 x2

x > 0.

sustituyendo x1 que es T 1 : (y1 , y2 ) (x1 , x2 )

J =

x1 y1 x2 y1

x1 y2 x2 y2

y2 1+y2 1 1+y2

y1 1+y2

y1 y2 (1+y2 )2

y1 (1+ y

2)

y1 . = (1 + y2 )2

Entonces fY1Y2 (y1 , y2 ) = donde y1 , y2 > 0; y1 y1 y1 y2 fX X , ; 1 2 2 (1 + y2 ) 1 + y2 1 + y2

y donde fX1X2 (x1 , x2 ) = ex1 ex2 .

Por tanto, la funci on de densidad conjunta buscada es: y1 fY1Y2 (y1 , y2 ) = 2 y1 ey1 2 (1 + y2 ) As pues, las distribuciones marginales son: Z fY1Y2 (y1 , y2 ) dy2 = 2 y1 ey1 , fY1 (y1 ) =
0

y1 > 0

y Z fY2 (y2 ) =
0

fY1Y2 (y1 , y2 ) dy1 =

y1 , (1 + y2 )2

y2 > 0.

Y puesto que fY1 (y1 ) fY2 (y2 ) = fY1Y2 (y1 , y2 ) ;

Y1 e Y2 son independientes.

12. Distribuciones multivariantes

Distribuciones conjuntas multivariantes

80

X = (X1 , X2 , . . . , Xn );

x = (x1 , . . . , xn ) Rn x RX1 RXn

FX (b) FX1 ,...,Xn (b1 , . . . , bn ) = P (X1 b1 , X2 b2 , . . . , Xn bn )

fX (x) fX1 ,...,Xn (x1 , . . . , xn )

fX (x) =

n Fx (x) ; x1 xn

b1

bn

Fx (b1 , . . . , bn ) =

fX (x) dxn dx1

Distribuciones marginales y marginales conjuntas

81

FXj (xj ) = FX1 ,...,Xn (, . . . , , xj , , . . . , )

fXj (xj ) =

fX (x) dxn dx(j +1) dx(j 1) dx1

RX1 RX(j 1) RX(j +1) RXn

fX2 Xn (x2 , . . . , xn ) =
RX1

fX (x) dx1

Distribuciones condicionales y esperanza condicional

82

Funci on de densidad condicionada de X1 f X1|X2 ...Xn ( x1 | b2 . . . bn) = fX (x1 , b2 , . . . , bn ) fX2 ...XN (b2 . . . bn )

Funci on de regresi on de X1 sobre X2 . . . Xn E X1|X2 ...Xn ( x1 | x2 . . . xn) = h(x2 , . . . , xn )

Independencia

83

f X1|X2 ...Xn ( x1 | x2 . . . xn) = fX1 (x1 ) ; por tanto


n

fX (x) =
i=1

fXi (xi ) .

13. Funci on generatriz de momentos

Funci on generatriz de momentos: caso univariante

84

Llamamos funci on generatriz de momentos de X a: MX (t) E etX

Caso discreto M (t) =


xRX

etx P X (x)

Caso continuo M (t) =


RX

etx fX (x) dx

siempre que dicha esperanza exista para t (h, h), donde h > 0.

Funci on generatriz de momentos: caso univariante

85

Puesto que d dM (t) = E etX = E dt dt d tX e dt = E XetX

entonces

dM (t) dt

= E (X )
t=0

Del mismo modo dM 2 (t) dt2

= E X2 ;
t=0

dM 3 (t) dt3

= E X3 ; . . .
t=0

Ejemplo 6. Sea fX (x) = ex , x > 0 entonces Z Z MX (t) =E etX = etx fX (x) dx = etx ex dx 0 0 Z 1 (t1)x = 0 1 = (1 t)1 e = e(t1)x dx = t1 t1 0 0 donde t debe ser menor que uno para que la integral est e denida. Por lo tanto su esperanza es MX (t) = y ` 2 MX (0) = 2 = E X 2 (1 t)3 ` 2 s pues, su varianza es Var (X ) = E X 2 E (X ) = 2 12 = 1. MX (t) = 1 MX (0) = 1 = E (X ) (1 t)2

Ejercicio 36. La funci on generatriz de momentos de una Chi-cuadrado de r grados de libertad es: M (t) = (1 2t)r/2 . Use su funci on generatriz de momentos para calcular la varianza de una Chi-cuadrado de r grados de libertad. 2 ` Soluci on: Puesto que Var (X ) = E X 2 E (X ) ; realizamos los siguientes c alculos: Por una parte: M (t) = que evaluada en cero es: E (X ) = M (0) = r Por otra parte: M (t) = r(
r r r 1)(1 2t) 2 2 (2) = (r2 + 2r)(1 2t) 2 2 2 r r r (1 2t) 2 1 (2) = r(1 2t) 2 1 2

que evaluada en cero es: ` E X 2 = M (0) = r 2 + 2r. Por tanto, Var (X ) = r 2 + 2r r 2 = 2r Ejercicio 36

Ejercicio 37. [Funci on generatriz de momentos de transformaciones lineales:] Sea X con MX (t), y sea Y = aX + b. (a) Demuestre que MY (t) = ebt MX (at). Soluci on : MY (t) = E eYt = E e(aX +b)t = E eaXt ebt = ebt E eaXt = ebt MX (at). Ejercicio 37 (b) Empleando el resultado anterior demuestre que E (Y ) = b + aE (X ) . Soluci on : MY (t) = dM (t) = bebt MX (at) + ebt aMX (at) dt MX (0) +a MX (0) | {z } | {z } E(X ) E (eX 0 )=E(1)=1

E (Y ) =MY (0) = b

= b + a E (X ) Ejercicio 37

(c) Sea Z N (0 , 1), su funci on generatriz de momentos es MZ (t) = et


2

/2

(v ease Novales, 1997, pp. 223); y sea Y = + Z. Demuestre que MY (t) = et+t
2

2 /2

Soluci on: Puesto que de (a) sabemos que MY (t) = et MZ (t); sustituyendo tenemos: MY (t) = et MZ (t) = et e(t)
2

/2

= et+

2 2

t /2

. Ejercicio 37

Funci on generatriz de momentos multivariante

86

Llamamos funci on generatriz de momentos de X = (X1 , X2 , . . . , XN ) evaluada en t = (t1 , t2 , . . . , tN ) a: MX (t) = MX (t1 , t2 , . . . , tN ) = E et1 X1 +t2 X2 ++tN XN discreto MX (t) = continuo MX (t) =
RX1

x1 RX1 xN RXN

et1 x1 ++tN xN P X1 ...XN (x1 , . . . , xN )

RXN

et1 x1 ++tN xN fX1 ...XN (x1 , . . . , xN ) dxN . . . dx1

Funci on generatriz de momentos

87

Los momentos son r+s MX (0) = E (Xi r Xj s ) s tr i tj La funci on generatriz marginal es

MXj (tj ) = MX (0, . . . , 0, tj , 0, . . . , 0) Adem as, si (X1 , X2 , . . . , XN ) son independientes, entonces

MX (t) =
i=1

MXi (ti )

Funci on generatriz de momentos: Ta de la unicidad

88

Teorema 13.1. Dos conjuntos de v.a., X e Y , tienen id entica distribuci on de probabilidad, si sus funciones generatrices de momentos coinciden (siempre que estas existan) y viceversa. fX (x) = fY (y ) MX (t) = MY (t)

Funci on generatriz de momentos: Suma de dos v.a. indep.

89

Sean X e Y independientes con MX (t) y MY (t) respectivamente. Entonces Z = X + Y tiene MZ (t) = MX (t) MY (t)

Demostraci on MZ (t) = E etZ = E etX +tY = E etX etY = por independencia = E etX E etY = MX (t) MY (t)
La demostraci on es similar para el caso de n v.a.s independientes.

Ejercicio 38. Sean X e Y variables aleatorias independientes con funci on generatriz de momentos (1 t2 )1 (distribuci on de Laplace) y con esperanza nula. (a) Sea U = X +Y . Demuestre que la funci on generatriz de momentos de U es (1t2 )2 . Soluci on : MU (t) =E et(X +Y ) = E etX E etY =MX (t) MY (t) = (1 t2 )2 Ejercicio 38 (b) Sea V = X Y . Demuestre que U y V tienen id entica distribuci on. Soluci on : MV (t) =E et(X Y ) = E etX E etY =MX (t) MY (t) = (1 t )
2 2

por independencia

por independencia

Por tanto tienen id entica distribuci on debido al teorema de unicidad de las funciones generatrices de momentos. Ejercicio 38

(c) Calcule la funci on generatriz de momentos conjunta de U y V Soluci on : MUV (s, t) =E esU +tV = E es(X +Y )+t(X Y ) =E e(s+t)X +(st)Y =E e(s+t)X E e(st)Y =MX (s + t) MY (s t) =(1 (s + t)2 )2 (1 (s t)2 )2 Ejercicio 38 (d) Demuestre que U y V no son independientes Soluci on: Y puesto que la funci on generatriz conjunta MU,V (s, t) es distinta de MU (s)MV (t) = (1 s2 )2 (1 t2 )2 U y V no son independientes. Ejercicio 38

por independencia

(e) No obstante, demuestre que U y V est an incorreladas ` ` Pista. recuerde que E (X ) =E (Y ) =0, y que E X 2 = E Y 2 puesto que ambas variables tienen id entica distribuci on. Soluci on: Basta con demostrar que Cov (U, V ) = 0. Cov (U, V ) = E (UV ) E (U ) E (V ) . Por una parte, E (U ) =E (X + Y ) = E (X ) + E (Y ) = 0 + 0 E (V ) =E (X Y ) = E (X ) E (Y ) = 0 0; por otra ` ` ` E (UV ) = E ((X + Y )(X Y )) = E X 2 Y 2 = E X 2 E Y 2 = 0. Por tanto Cov (U, V ) = 0. Ejercicio 38

Ejercicio 39. Sea la siguiente funci on de cuant a PXY (x, y ) x=0 x=1 y=0 1/ 9 1/18 y=1 3 /9 1 /6 y=2 2/9 1/9

(a) Obtenga la funci on generatriz de momentos conjunta. ` Soluci on: Puesto que MX,Y (t, ) = E etX + Y ; entonces X X MX,Y (t, ) =E etX + Y = etxi + yj PXY (xi , yj )
xi RX yj RY

=1/9e0t+0 + 3/9e + 2/9e2 + 1/18et + 1/6et+ + 1/9et+2 Ejercicio 39

(b) Utilizando exclusivamente la funci on generatriz de momentos calculada en el ejercicio anterior obtenga la esperanza de X . Soluci on: La funci on generatriz de momentos marginal es la conjunta evaluada en = 0, por tanto MX (t) = 2/3 + 1/18et + 1/6et + 1/9et = 2/3 + 1/3et derivando respecto de t y evaluando en t = 0 d (2/3 + 1/3et ) E (X ) = = 1/3e0 = 1/3. dt t=0 Ejercicio 39

14. Preguntas y problemas

Test. Conteste a las siguientes cuestiones. 1. Sean X e Y dos variables aleatorias tales que Y = 2X + 1. Adem as, se tiene que la variable X sigue una distribuci on uniforme en el intervalo [1, 1]. Se tiene que: (a) = 1, donde es el coeciente de correlaci on entre X e Y . (c) Var (Y |X = 0) = Var (Y ). (b) E Y|X ( Y | x) = E (Y ). (d) E Y|X ( Y | x) = E X|Y ( X | y)

2. Suponga que E X|Y ( X | y) = E (X ) para todo y , entonces necesariamente: (a) conocer el valor tomado por Y no aporta informaci on sobre la probabilidad de los valores que puede tomar X . (b) saber que Y ha tomado un valor superior a E (Y ) no aporta informaci on sobre la probabilidad de que X tome un valor superior a E (X ). (c) E Y|X ( Y | x) = E (Y ) para todo x. (d) Var X|Y ( X | y) = Var (X ) para todo y . Soluci on : conocer el valor de Y no aporta informaci on sobre los valores que puede haber olo cuando X e Y son independientes; algo que no necesariamente tomado X s ocurre cuando E X|Y ( X | y) = E (X ). Puesto que la esperanza condicionada de X no depende de los valores tomados por Y , saber que Y ha tomado un valor determinado no nos dice nada acerca de E (X ) (por lo tanto 2 es cierta). En general, E X|Y ( X | y) = E (X ) no implica ni E Y|X ( Y | x) = E (Y ), ni tampoco Var X|Y ( X | y) = Var (X ). Fin 2

El siguiente texto es valido para las 3 siguientes preguntas: Sea la funci on k si 0 < x < y < 2 fXY (x, y ) = 0 resto 3. El valor de k que hace que fXY (x, y ) sea funci on de densidad viene dado por la ecuaci on: R2R2 R2R2 kdydx = 1. (a) x (b) 0 x kdydx = 1. R 2 R0 2 kdydx = 1. (c) 0 0 (d) ninguna de las anteriores.

4. Conocida una realizaci on X = 1, se tiene que: (a) (b) (c) (d) la realizaci on de Y est a en el intervalo (0, 1). para cualesquiera realizaciones, las variables X e Y son independientes. E Y|X ( Y | 1) < E (Y ). E Y|X ( Y | 1) E (Y ).

5. Se tiene que: (a) (b) (c) (d) R 1 R 2x fXY (x, y ) dydx. P (X + Y 2) = 0 R 1 Rx 1 f P (X + Y 2) = 0 (x, y ) dydx. XY R 1 R0 2 f P (X + Y 2) = 0 (x, y ) dydx. XY x ninguna de las anteriores. n del grupo de preguntas

La siguiente funci on se utiliza en las preguntas 6, 7, 8, 9 y 10: Sean X e Y variables aleatorias con funci on de densidad conjunta (para alg un valor concreto de k) k[1 + xy (x2 y 2 )] para 1 < x < 1; 1 < y < 1 fXY (x, y ) = . 0 en los restantes casos 6. Que valor debe tomar k para que fXY (x, y ) sea funci on de densidad. (a) k = 4 (c) k = 2 Soluci on : 1= = R1 R1
1 1

(b) k = 1/4 (d) k = 1/2 k[1 + xy (x2 y 2 )]dxdy dxdy + R1 R1


1 1 (x 3y

hR R 1 1
1 1

xy 3 )dxdy

R 1 h x4 k 4+ y 1 4 R 1 hy k 4+ 1 4
y3 2

i1 x2 3 y 2 1
y 4

dy dy = k 4 Fin 6

= por tanto k = 1/4.

i1 y3 2 1

7. Las funciones de densidad marginal de X y Y son (a) (b) (c) (d) fX (x) = 1/2; fX (x) = 1/2; fX (x) = fX (x) =
1 2 1 2

fY (y ) = 1/4 fY (y ) = 1/2
x3 2 x3 2

+ +

x ; 4 x ; 4

fY (y ) = fY (y ) =

1 4 1 2

+ +

y 4 y 4

y2 2 y2 2

Soluci on:La densidad marginal de X es Z 1 1 2 x3 x x3 x 1 fX (x) = [1 + xy (x2 y 2 )]dy = + + = 4 2 4 2 4 2 1 4 De manera an aloga fY (y ) =


1 2

Fin 7

8. La covarianza entre X e Y es (a) (b) (c) (d) Cov (X, Y ) = 1 5 Cov (X, Y ) =0 Cov (X, Y ) =0; y por lo tanto X e Y son independientes ninguna de las anteriores

Soluci on:Podemos calcular Cov (X, Y ) como Cov (X, Y ) = E (XY ) E (X ) E (Y ) . Por una parte Z E (X ) = 1 1 x dx = 2 2 1
1

x2 2

1 = 0,
1

y an alogamente E (Y ) = 0. Por otra parte R1 R1 1 2 2 E (XY ) = 1 1 xy 4 [1 + xy (x y )]dxdy 2 =


1 4

7 6 1 1 1 R 1 1 R 1 R R 7 6R R xydxdy + x4 y 2 dxdy x2 y 4 dxdy 7 6 41 1 5 1 1 1 1 | {z } | {z } | {z }


A B C

donde Z1 Z1 A=
1 1

Z1 xydxdy =
1

Z1 xdx
1

ydy = 0,

Z1 Z1 B=
1 1

Z1 xydxdy =
1

x4 dx

Z1

y 2 dy =

2 2 4 = , 5 3 15 2 2 4 = , 5 3 15

Z1 Z1 C=
1 1

Z1 xydxdy =
1

x2 dx

Z1

y 4 dy =

por lo tanto E (XY ) = Podemos concluir que Cov (X, Y ) = E (XY ) E (X ) E (Y ) = 0 0 0 = 0; es decir, no hay relaci on lineal entre X e Y , lo que no quiere decir no pueda existir alg un otro tipo de relaci on; de hecho 11 1 1 fX (x) fY (y ) = = = fXY (x, y ) = [1 + xy (x2 y 2 )] 22 4 4 por lo que ambas variables no son independientes. Fin 8 1 [A + B C] = 0. 4

9. Se nale la armaci on correcta. La esperanza de Y condicionada a X = x es: (a) E Y|X ( Y | x) = 0 (c) E Y|X ( Y | x) > E (Y ) x3 1 x (b) E Y|X ( Y | x) = 1 3 5 (d) E Y|X ( Y | x) < E (Y )
fXY (x,y ) fX(x)
1 [1+xy (x2 y 2 )] 4

Soluci on:Necesitamos la funci on de densidad condicional, es decir f Y|X ( Y | x) = = = La esperanza condicional es Z E Y|X ( Y | x) =
1 1

1/2 1 [1 2

+ xy (x2 y 2 )]

1 1 1 y [1 + xy (x2 y 2 )] = x3 x. 2 3 5 Fin 9

10. Cual es la probabilidad de que X + Y 0? (a) 5/24 (c) 1/4 Soluci on : P (X + Y > 0) = 1 [1 + xy (x2 y 2 )]dydx 4 y=1 Z 1 y x3 y 2 xy 4 + dx = 2 4 y=x 1 4 Z 1 1 x3 x x 2 x5 x5 = + + dx 4 2 4 4 4 4 1 1 Z 1 1 x3 x5 x x4 x6 = + dx = + 2 4 4 8 24 1 1 4 1 1 1 1 1 1 1 = + + = 4 8 24 4 8 24 2
1 x

(b) 1/2 (d) 3/8 Z


1

Fin 10

n del grupo de preguntas

La siguiente informaci on es v alida para las dos pr oximas preguntas: Suponga la siguiente tabla de probabilidades de la variable aleatoria bidimensional (X ,Y ): Y=0 Y=1 X=1 2/18 1/18 X=2 6/18 3/18 X=3 4/18 2/18

11. De las siguientes armaciones: (i) PX (X = 2) = 9/18 (ii) PY (Y = 0) = 12/18 (iii) E (X ) = 13/6 (iv) Su coeciente de correlaci on lineal es cero S olo son ciertas: (a) La (i) y la (ii) (b) Todas (c) Todas menos la (iv) (d) Ninguna

12. Cu al de las siguientes armaciones es verdadera? (a) E Y|X ( Y | 2) = 2/3 (c) E Y|X ( Y | 2) > E (Y ) (b) E Y|X ( Y | 2) = E Y|X ( Y | 3) (d) E Y|X ( Y | 2) < E (Y ) n del grupo de preguntas

13. Sean X e Y dos variables aleatorias tales que Y = aX 2 , donde a > 0 es constante y x < 0. Cu al de las siguientes armaciones es correcta? (a) El coeciente de correlaci on lineal es 1 al ser a > 0. (b) El coeciente de correlaci on lineal no tiene porque ser 1, pero siempre ser a positivo al ser a > 0. (c) El coeciente de correlaci on lineal es cero debido al t ermino cuadr atico. (d) El coeciente de correlaci on lineal depende de la ley de probabilidades de X .

14. Sean X e Y dos variables aleatorias tales que Y = aX + b y Var (X ) > 0, donde a y b son n umeros jos. Sea el coeciente de correlaci on poblacional entre ambas variables. Se tiene que: (a) = 1 si |a| =1 ` (c) Var Y X = 1 > Var (Y ) (b) = 1 para cualquier a (d) | | = 1 si a = 0

Soluci on : Si a = 1; |a| = 1 y sin embargo = 1 Si a < 0, = 1. ` Dada la relaci on determinista entre X e Y , se verica que Var Y X = 1 = 0 ya que si X = 1 entonces ( Y | X = 1) = a + b es una constante; pero si a = 0 entonces Var (X ) > 0, que implica que Var (Y ) > 0. Fin 14

La siguiente funci on se utiliza en las preguntas 15, 16 y 17: k si 0 < x < 2y < 1 fXY (x, y ) = 0 resto 15. El valor de k que hace que fXY (x, y ) sea funci on de densidad viene dado por la ecuaci on: R 1 R 2x (a) 0 0 fXY (x, y ) dydx = 1. (b) (c) R
1x 2

R1
0

fXY (x, y ) dxdy = 1.

R 1 R 2y
0 0

fXY (x, y ) dydx = 1.

(d) ninguna de las anteriores. Soluci on:El soporte es el tri angulo con v ertices (0, 0), (0, 1/2) y (1,1/2).
y
x= 2y x/2 y=

1/2 y0 0 x0 1

Por lo tanto la respuesta correcta es Z 1 Z 1/2 fXY (x, y ) dydx = 1.


0 x/2

o bien
y
x= 2y x/2 y=

1/2 y0 0 x0 1

Z
0

1/2

Z
0

2y

fXY (x, y ) dxdy = 1. De hecho el valor de k es Z 1/2 Z 2y Z kdxdy = k


0 0 0

1/2

1/2 2ydy = k y 2 0 = k 1/4 = 1 Fin 15

por tanto k = 4.

16. Conocida una realizaci on de X =

se tiene que: ` 1 (a) la realizaci on de Y est a en el intervalo 0, 4 . (b) sabemos lo mismo sobre las posibles realizaciones de Y que si no conoci eramos la realizaci on de X . (c) esperamos que Y haya tomado un valor inferior a E (Y ). (d) esperamos que Y haya tomado un valor igual o superior a E (Y ).

1 , 2

Soluci on : Puesto que X = 1/2 y x < 2y < 1 la realizaci on de y debe pertenecer al intervalo (1/4, 1/2). La distribuci on marginal de X es Z 1/2 Z 1/2 1/2 fXY (x, y ) dy = 4dy = 4y x/2 = 2 2x
x/2 x/2

y la de Y es Z
0 2y

2y 4dx = 4x 0 = 8y

De donde deducimos que en general fX (x) fY (y ) = 8y (2 2x) = 4 = fXY (x, y ) por tanto no son independientes y la informaci on de X condiciona la distribuci on de Y .

Por una parte E (Y ) = = por otra f Y|X ( Y | x) = por lo que E Y|X ( Y | x) = = = As pues, ` E Y X = 1/2 = y2 1/ 2 1/2 = 0.375 > 1/3 = E (Y ) .
1/4

R 1/2
0

yfY (y ) dy =

R 1/2
0

y 8ydy =

8y3 1/2
3 0

1/3; fXY (x, y ) 4 2 = = fX (x) 2 2x 1x R 1/2


2 Rx/ 1/2

yf Y|X ( Y | x) dy

y 2 dy x/2 1x y 2 1/2 . 1x x/2

Fin 16

17. Sea a ( 1 , 3 ) y Z = X + Y , se tiene que: 2 2 R a 1 R1 2 2 (a) P (Z a) = 1 fXY (x, y ) dydx 0 P (Z a ) = 1 x fXY (x, y ) dydx 0 2 RaRa (c) P (Z a) = 0 0 fXY (x, y ) dydx (d) ninguna de las anteriores. (b) Soluci on :
y
1 R R a 2 a

x 2

+ +

R R

2a 3

R ax
1x 2 1x 2

a 1 2
2a 3 a 1 2

fXY (x, y ) dydx fXY (x, y ) dydx

R ax

1/2
y = a x


1 x
x= 2y x/2 y=

0 a

2a 1 3 2

Fin 17

18. Sean X1 , X2 , . . . , Xn i.i.d con momentos bien denidos. Se nale la armaci on b el estimador de media muestral. correcta. Sea x b coincide con la funci (a) La funci on generatriz de x on generatriz de momentos conjunta de X1 , X2 , . . . , Xn evaluada en (1/n, 1/n, . . . , 1/n). (b) Si las variables aleatorias Xi tienen distribuci on normal, entonces necesitamos aplicar el Teorema Central del L mite para conocer la distribuci on asint otica de b. x b coincide con E (X ) s (c) La esperanza de x olo si las variables aleatorias Xi tienen distribuci on normal. (d) Si variables aleatorias Xi no tienen distribuci on normal, no podemos conocer de b ni su distribuci x on para muestras nitas, ni tampoco su distribuci on asint otica.

Ejercicio 40. Sea X una variable aleatoria con la siguiente funci on generatriz de momentos MX (t) = (1 + t2 ). (a) Calcule E (X ) Soluci on : dM (t) = E (X ) = 20 = 0 dt t=0 Ejercicio 40 (b) Calcule Var (X ) Soluci on: Puesto que ` dM 2 (t) = E X 2 = 2 dt2 t=0 ` Entonces Var (X ) = E X 2 [E (X )]2 = 2

Ejercicio 40

Ejercicio 41. Comente y/o responda brevemente las siguientes frases:

(a) En ning un caso podemos concluir que dos variables aleatorias que est en incorrelacionadas sean independientes. Soluci on: Cuando ambas variables aleatorias tienen distribuci on conjunta normal, entonces incorrelaci on implica necesariamente independencia. Ejercicio 41 (b) Tenemos dos variables X , Y . Su coeciente de correlaci on lineal est a muy pr oximo a cero. As , concluimos que la la relaci on entre las dos variables es pr acticamente inexistente. Soluci on: La conclusi on es falsa. La correlaci on lineal indica el grado de relaci on lineal entre dos variables. Si el coeciente de correlaci on lineal est a muy pr oximo a cero, esto quiere decir que relaci on lineal entre las dos variables es pr acticamente inexistente. Pero esto no es obice para que pueda existir una fuerte relaci on (no lineal) entre las variables. Ejercicio 41 (c) Dadas dos variables aleatorias X , Y . Sus momentos de primer y segundo orden existen, est an bien denidos y son conocidos. La aproximaci on lineal a la esperanza 2 . condicional E Y|X ( Y | x) es a + b x, siendo a = E(Y ) bE(X ) y b = Cov(X, Y ) /X Pese a su sencillo c alculo, esta aproximaci on tiene el inconveniente de que es siempre una mera aproximaci on (es decir, nunca podemos garantizar que coincida con la verdadera esperanza condicional). Soluci on: Cuando la esperanza condicional es una funci on lineal de x, la aproximaci on

lineal coincide con la verdadera esperanza condicional (esto siempre es as cuando X e Y tienen distribuci on conjunta normal). Ejercicio 41

15. Trasparencias Lista de Trasparencias


1 Modelo Te orico vs Mundo Real 2 Regularidades en el azar 3 Lanzamiento de dos dados 4 Lanzamiento de dos dados 5 Regularidades en fen omenos econ omicos? 6 Indice NASDAQ 7 Tasa de Variaci on del Indice NASDAQ 8 Tasa de Variaci on del Indice NASDAQ 9 Estad stica y Econometr a 10 Probabilidad condicional 11 Independencia 12 Leyes de probabilidad conjunta y marginales 13 Leyes de probabilidad conjunta, marginales y condicionadas 14 Probabilidad conjunta y marginal 15 Concepto de Vble. aleatoria 16 Funci on de cuant a y funci on de distribuci on 17 Funci on de cuant a: Distribuci on Bernulli 18 Funci on de distribuci on: Distribuci on Bernulli

19 20 21 22 23 24 25 26 27 28 29 30 31 32 33 34 35 36 37 38 39

Funci on de densidad y funci on de distribuci on Funci on de densidad Funci on de densidad: Distribuci on Normal (Gaussiana) Variables aleatorias: Una gran simplicaci on! Variables aleatorias: Una gran simplicaci on! De (S, B, P ()) a un modelo de probabilidad: historia en s mbolos Modelado emp rico Vector aleatorio Vector aleatorio: funci on de distribuci on Variables discretas: Funci on de cuant a conjunta Funci on de cuant a: ejemplo Funci on de cuant a conjunta: ejemplo Funci on de cuant a: Bivariante Funci on de distribuci on y funci on de cuant a conjuntas Funci on de distribuci on: Bivariante Variables continuas: Funci on de densidad conjunta Funci on de densidad conjunta Propiedades de las leyes de probabilidad bivariantes Distribuciones conjuntas y marginales: ejemplo Funci on de cuant a marginal Distribuciones conjuntas y marginales

40 41 42 43 44 45 46 47 48 49 50 51 52 53 54 55 56 57 58 59 60

Distribuciones conjuntas y marginales Distribuciones conjuntas y marginales Funciones de cuant a condicionadas Probabilidad conjunta, marginal y condicionada Funci on de cuant a condicionada Distribuciones de probabilidad condicionadas: Caso continuo Distribuciones condicionadas Independencia Momentos conjuntos: caso discreto Momentos conjuntos: caso cont nuo Momentos te oricos bivariantes: propiedades Covarianza Coeciente de correlaci on Momentos condicionados: caso discreto Momentos condicionados: caso continuo Propiedades de la esperanza condicional Esperanza condicional como funci on de x: E Y|X ( Y | x) Esperanza condicional como v.a.: E (Y | X ) Varianza condicional como funci on de x: Var Y|X ( Y | x) Varianza condicional como v.a.: Var (Y | X ) Teorema de las esperanzas iteradas

61 62 63 64 65 66 67 68 69 70 71 72 73 74 75 76 77 78 79 80 81

Identidad de la varianza condicional Propiedades de la esperanza condicional estoc astica Esperanza condicional como predictor optimo Esperanza condicional como predictor optimo Aproximaci on lineal a la esperanza condicional: Recta de regresi on Aproximaci on lineal a la esperanza condicional: Recta de regresi on Aproximaci on lineal a la esperanza condicional: Recta de regresi on Aproximaci on lineal a la esperanza condicional: Recta de regresi on Aproximaci on lineal a la esperanza condicional: Recta de regresi on Transformaci on de una v.a. unidimensional Transformaci on de una v.a. unidimensional Transformaci on de una v.a. unidimensional Transformaci on de una v.a. unidimensional Transformaci on de una v.a.: ejemplo Transformaci on de una v.a.: ejemplo Esp. condicional como funci on de regresi on o como v.a.: utilidad Esp. condicional como funci on de regresi on o como v.a.: utilidad Fun. de densidad de la esperanza condicional: suponiendo linealidad Transformaci on de una v.a. bidimensional Distribuciones conjuntas multivariantes Distribuciones marginales y marginales conjuntas

82 83 84 85 86 87 88 89 90 91 92 93

Distribuciones condicionales y esperanza condicional Independencia Funci on generatriz de momentos: caso univariante Funci on generatriz de momentos: caso univariante Funci on generatriz de momentos multivariante Funci on generatriz de momentos Funci on generatriz de momentos: Ta de la unicidad Funci on generatriz de momentos: Suma de dos v.a. indep. Propiedades de la esperanza Propiedades de la varianza P (X = x): caso continuo Partes del temario

16. Bibliograf a
Casella, G. y Berger, R. L. (2002). Statistical Inference . Duxbury Advanced Series. Duxbury, USA, segunda ed. ISBN 0-534-24312-6. 279 L opez Cachero, M. (1992). Fundamentos y m etodos de estad stica. Ediciones Pir amide, Madrid. ISBN 84-368-0425-2. 21 Mittelhammer, R. C. (1996). Mathematical Statistics for Economics and Business . Springer-Verlag, New York, primera ed. ISBN 0-387-94587-3. 21

Novales, A. (1997). Estad stica y Econometr a. McGraw-Hill, Madrid, primera ed. ISBN 84-481-0798-5. 7, 21, 44, 46, 125, 154, 205, 233 Papoulis, A. (1991). Probability, random variables, and stochastic processes . Electrical & Electronic Engineering Series. McGraw-Hill, New York, tercera ed. ISBN 0-07-100870-5. 202 Pe na, D. (2001). Fundamentos de Estad stica. 84-206-8696-4. 7, 21 Alianza Editorial, Madrid. ISBN

Pe na, D. (2002). Regresi on y dise no de experimentos . Alianza Editorial, Madrid. ISBN 84-206-8695-6. 7 Pe na, D. y Romo, J. (1997). Introducci on a la Estad stica para la Ciencias Sociales . McGraw-Hill, Madrid. ISBN 84-481-1617-8. 7 Spanos, A. (1999). Probability Theory and Statistical Inference. Econometric Modeling with Observational Data. Cambridge University Press, Cambridge, UK. ISBN 0-521-42408-9. 9, 12, 21

A. Momentos univariantes

Propiedades de la esperanza

90

Para X1 y X2 v.a. y a, b y c constantes; E () satisface las siguientes propiedades: E1. E (c) = c E2. E (aX1 + bX2 ) = aE (X1 ) + bE (X2 )

Por lo tanto, la aplicaci on esperanza, E (), es lineal.

Propiedades de la varianza

91

Para X1 y X2 v.a. independientes ; y a, b y c constantes; Var () satisface las siguientes propiedades: V1. Var (c) = 0 V2. Var (aX1 + bX2 ) = a2 Var (X1 ) + b2 Var (X2 )

B. Demostraciones

P (X = x): caso continuo

92

Puesto que {X = x} {x < X x} para cualquier

> 0, entonces

P (X = x) P (x < X x) = FX (x) FX (x ) para cualquier > 0. Por tanto


0

0 P (X = x) l m [FX (x) FX (x )] = 0 por continuidad de FX (x) . (Casella y Berger, 2002, pp. 35)

Partes del temario


Tema 1 IntEctr-T01 Tema 2 IntEctr-T02 Tema 3 IntEctr-T03 Tema 4 IntEctr-T04 Tema 5 IntEctr-T05 Tema 6 IntEctr-T06 Tema 7 IntEctr-T07

Soluciones a los Ejercicios Ejercicio 33(a)


PXY (x, y ) Y = 1 Y=0 Y=1 que suma 9 k; por tanto k =
1 . 9

X = 1 0k 1k 2k

X=0 1k 1k 1k

X=1 2k 1k 0k

Ejercicio 33(b) Primero calculamos la funci on de cuant a marginal P X ( x) Por tanto E (X ) = X xP X (x) = X = 1 1/3 X=0 1 /3 X=1 1/3

1 1 =0 3 3

Ejercicio 33(c) Primero necesitamos calcular la funci on de cuant a condicionada a que Y = 1; sabiendo que P (Y = 1) = 1 3 P X|Y ( x | 1) Por tanto E X|Y ( X | 1) = X xP X|Y ( x | 1) = X = 1 0 X=0 1 /3 X=1 2/3 2 3

Ejercicio 33(d) Por una parte ` X 2 1 1 2 E X2 = x P X ( x) = + = 3 3 3 X 1 1 =0 3 3 ` 2 Var (X ) =E X 2 E (X )2 = 3

E (Y ) =

yP Y (y ) =

` X 2 1 1 2 E Y2 = y P Y (y ) = + = 3 3 3

` 2 Var (Y ) =E Y 2 E (Y )2 = 3 XX 2 4 2 = 9 9 9

E (XY ) = Por tanto

xyP Y (y ) =

Cov (X, Y ) =E (XY ) E (X ) E (Y ) =

4/ 9 4/9 2 XY = p = = 2/3 3 2/ 3 2/3

Ejercicio 33(e) Z es una variable cuyo soporte son los valores enteros desde 2 hasta 2. Pensemos primero cuales son sus probabilidades: P (Z = 2) =PXY (1, 1) = 2 9 1 2 1 + = 9 9 9 1 1 +0= 9 9

P (Z = 1) =PXY (1, 0) + PXY (0, 1) =

P (Z = 0) =PXY (1, 1) + PXY (0, 0) + PXY (0, 0) = 0 + P (Z = 1) =PXY (0, 1) + PXY (1, 0) = P (Z = 2) =PXY (1, 1) = 2 9 1 1 2 + = 9 9 9

Por tanto E (Z ) = 2

2 2 1 2 2 1 0+ +1 +2 =0 9 9 9 9 9

Ejercicio 33(f) (Z | Y = 1) es una variable cuyo soporte son los valores enteros desde 0 hasta 2. Pensemos primero cuales son sus probabilidades: P ( Z = 0 | Y = 1) = P X|Y ( 1 | 1) =0 P ( Z = 1 | Y = 1) = P X|Y ( 0 | 1) = 1 3 2 P ( Z = 2 | Y = 1) = P X|Y ( 1 | 1) = 3

Por tanto E Z|Y ( Z | 1) = 1

1 2 5 +2 = 3 3 3

Ejercicio 33(g) E Y|X ( Y | x) 0 + Cov (X, Y ) 2 x= x Var (X ) 3

S-ar putea să vă placă și